Peds final (ch 1, 3-7, 13-15, 18-21, 23): part 2 (fixed)

¡Supera tus tareas y exámenes ahora con Quizwiz!

Which type of medication lacks a ceiling effect, and therefore is prescribed in initial doses that must be titrated to achieve pain relief while managing side effects?

morphine

Which condition is a risk factor for the development of pelvic inflammatory disease? -multiple sexual partners -oral contraceptive use -recurrent urinary infections -history of dysmmenorrhea

multiple sexual partners Clients who have had multiple sexual partners have a higher incidence of developing pelvic inflammatory disease. Oral contraceptive use, history of UTI, and dysmmeorrhea are not risk factors for developing pelvic inflammatory disease.

The nurse is reinforcing teaching with the caregiver of 5-year-old twins regarding urinary tract infections (UTIs). The caregiver is puzzled about why her daughter has had three urinary tract infections but her son has had none. She reports that their diets and fluid intake is similar. Which statement would be accurate for the nurse to tell this mother? -"A girl's urethra is much shorter and straighter than a boy's, so it can be contaminated fairly easily." -"Girls tend to urinate less frequently than boys, making them more susceptible to UTI's." -"Girls need more vitamin C than boys to keep their urinary tract healthy, so your daughter may be deficient in vitamin C." -"It is unlikely that your daughter is practicing good cleaning habits after she voids."

"A girl's urethra is much shorter and straighter than a boy's, so it can be contaminated fairly easily." Many different bacteria may infect the urinary tract, and intestinal bacteria, particularly Escherichia coli, account for about 80% of acute episodes. The female urethra is shorter and straighter than the male urethra, so it is more easily contaminated with feces.

A client's mother asks the nurse, "When should my daughter have a pelvic examination?" Which response by the nurse is most appropriate? -"A pelvic examination is not necessary until pregnancy." -"A pelvic exam is necessary at 18 to 20 years of age." -"A pelvic exam is necessary for girls in puberty." -"As her mother, it is your choice when she should have a pelvic exam."

"A pelvic exam is necessary at 18 to 20 years of age." A pelvic exam is unnescessary for girls who have not yet reached adolescence. A pelvic exam should be part of routine health care around the age of 18 to 20 years or at the point when she becomes sexually active.

The nurse is showing the student nurse how to flush a pediatric client's peripherally inserted central catheter (PICC) line. The nurse prepares a 3-mL normal saline flush using a 5-mL syringe. The student asks the nurse why the flush was prepared this way. What is the most accurate response by the nurse?

"Using a larger-volume syringe exerts less pressure on the PICC line." Explanation: Using a larger-volume syringe (i.e., 5 mL or larger) exerts less pressure on the PICC, thereby reducing the risk of rupture.

The nurse is caring for a newborn with a cleft palate. Which findings in the maternal medical record are considered to be contributing factors? Select all that apply. a) Moderate maternal alcohol use prior to pregnancy. b) Anticonvulsant therapy used to manage a seizure disorder. c) Reports of marijuana use in early pregnancy. d) Maternal tobacco use. e) Maternal age less than 18 years.

• Maternal tobacco use. • Anticonvulsant therapy used to manage a seizure disorder. Infants born with a cleft palate may have mother's with risk factors. These include maternal smoking, prenatal infection, advanced maternal age, use of anticonvulsants or steroids

A child is hospitalized with dehydration as a result of rotavirus. When reviewing the plan of treatment, what can the nurse anticipate will be included? Select all that apply. a) Antidiarrheal agents b) Monitor of intake and output c) Antibiotic therapy d) IV fluid administration e) Daily weight assessment

• Monitor of intake and output • IV fluid administration • Daily weight assessment Rotavirus is viral in nature. Antibiotic therapy is not used in the care and treatment of a viral infection. Antidiarrhea medications are not utilized as they are not effective. Intake and output will be observed. Daily weight will aid in the determination of hydration status. IV fluids may be indicated in the rehydration process.

The nurse is talking with a pregnant client about cleft lips and palates. The client has asked if these can be tested for. What information should be included in the nurse's response? Select all that apply. a) The nuchal translucency test can be used to screen for cleft lips and palates. b) The quadruple marker test can be used to detect these conditions. c) Most cleft lips and palates are found at delivery. d) Ultrasounds can be used to assess for these conditions. e) There are no ways to determine the presence of cleft lips or palates prior to delivery.

• Most cleft lips and palates are found at delivery. • Ultrasounds can be used to assess for these conditions. Ultrasounds can be used to identify the presence of cleft lips or palates. Most, however, are found after birth. The quadruple screening test assesses for potential down syndrome and neuro tube defects. Nuchal translucency testing is used to assess for down syndrome.

The nurse is positioning an infant who has just had his left-sided cleft lip repaired. What positions are acceptable for this infant? Select all that apply. a) High fowlers b) Supine c) Prone d) Right side lying e) Left side lying

• Right side lying • Supine It is critical to prevent injury to the facial suture line or to the palatal operative sites. Do not allow the infant to rub the facial suture line. To prevent this, position the infant in a supine or side-lying position

The nurse is reinforcing dietary teaching with the caregiver of a child diagnosed with celiac syndrome. Which foods would be permitted in the diet of the child with celiac syndrome? Select all that apply. a) Skim milk b) Rye bread c) Oatmeal d) Applesauce e) Corn flakes f) Bananas

• Skim milk • Applesauce • Bananas The child is usually started on a gluten-free, low-fat diet. Skim milk and bananas are usually well tolerated. Lean meats, puréed vegetables, and fruits are gradually added to the diet. Wheat, rye, oat products, corn flour, and cornmeal are not included in the diet.

Which instruction should a nurse give to a client who has a history of urinary tract infection to prevent recurrence? -Wipe from front to back. -Use bubble bath to wash. -Encourage fluids throughout the day. -Finish all antibiotic prescribed. -Limit bathing to once a week.

-Wipe from front to back. -Encourage fluids throughout the day. -Finish all antibiotic prescribed. Teaching caregivers to wipe from front to back, encouraging fluids, and finishing all prescribed medications are vital principles in the prevention of recurring UTIs. The use of bubble bath is contraindicated because it can be a source of infection.

A nurse is providing care for a child diagnosed with beta-thalassemia. The child requires a blood transfusion of packed red blood cells (PRBCs). The health care provider has prescribed a transfusion volume of 10 ml/kg. The child weighs 37 lb (16.8 kg). How many milliliters should the nurse infuse?

168 Explanation: The nurse will use the client's weight in kilograms and multiply by the prescribed milliliters/kilogram 16.8 kg × 10 ml PRBCs = 168 mLl

A nurse is providing care for a child diagnosed with beta-thalassemia. The child requires a blood transfusion of packed red blood cells (PRBCs). The health care provider has prescribed a transfusion volume of 10 ml/kg. The child weighs 37 lb (16.8 kg). How many milliliters should the nurse infuse?

168 Explanation: The nurse will use the client's weight in kilograms and multiply by the prescribed milliliters/kilogram 16.8 kg × 10 ml PRBCs = 168 mLl

The nurse is caring for a burn client with orders for oral ibuprofen and morphine PRN to control pain. Which nursing interaction is the most beneficial for the nurse to implement for pain management?

Alternate these medications around the clock to diminish peaks and valleys in pain control.

The nurse is doing dietary teaching with the caregivers of a child diagnosed with idiopathic celiac disease. Of the following foods, which would most likely be appropriate in the child's diet? a) Bananas b) Potatoes c) Oatmeal d) Toast

Bananas The young child should be started on a starch-free, low-fat diet. Bananas contain invert sugar and are usually well tolerated. Products that contain wheat, rye, or oats should be excluded.

A child with HIV, weighing 25 kg (55.1 lbs), is about to receive an infusion of IVIG. The recommended dose is 400 mg/kg/dose. The medication is available in a concentration of 50 mg/mL. What is the proper amount of infusion that the child will receive?

200 mL Explanation: The dose is calculated as 25 x 400 = 10,000 mg. Because the concentration is 50 mg/mL, calculate the volume as 10,000/50 = 200 mL.

A health care provider has prescribed hydroxyurea 20 mg/kg to a child as part of a treatment regimen for sickle cell disease. The child weighs 27 lb (12.2 kg). How many milligrams should the nurse administer?

244 Explanation: The nurse will use the client's weight in kilograms and multiply by the prescribed milligrams per kilogram. 12.2 kg × 20 mg/kg = 244 mg

The nurse is calculating the urinary output for the infant. The infant's diaper weighed 40 grams prior to placing the diaper on the infant. After removal of the wet diaper, the diaper weighed 75 grams. How many milliliters of urine can the nurse document as urinary output? Record your answer using a whole number.

35 Explanation: The diaper must be weighed before being placed on the infant and after removal to determine urinary output. For each 1 gram of increased weight, this is the equivalent of 1 milliliter of fluid. 75 grams - 40 grams = 35 grams = 35 mL

What are some negative effects that chronic pain can have on the pediatric population?

sleep disturbances, exhaustion, irritability, mood disturbances, and depression

A nurse is assessing the skin of a 12-year-old with suspected right ventricular heart failure. Where should the nurse expect to note edema in this child?

Lower extremities Explanation: Edema of the lower extremities is characteristic of right ventricular heart failure in older children. In infants, peripheral edema occurs first in the face, then the presacral region, and the extremities.

the nurse wishes to promote gastric emptying after administering the preschooler's gastrostomy feeding. Which position will facilitate this?

Right side-lying The right side-lying position should be chosen because the stomach empties into the intestine in this direction. It is also helpful to elevate the child's head slightly to prevent reflux of the feeding into the esophagus.

The infant is listless with sunken fontanels and has been diagnosed with dehydration. The infant is still producing at least 1 mL/kg each hour of urine. The infant weighs 13.2 lb (6 kg). At the minimum, how many milliliters of urine will the infant produce during the next 8-hour shift? Record your answer using a whole number.

48 Urine output should be calculated using weight in kilograms. 6 kg x 1 mL/kg = 6 mL/hour 6 mL x 8 hours = 48 mL/8-hour shift

The adolescent weighs 113 lb (51.36 kg). The nurse closely monitors the child's urine output. How many milliliters of urine is the least amount that the adolescent should make during an 8-hour shift? Record your answer using a whole number.

411 Explanation: The child weighs 113 lb (51.36 kg). 51.36 kg x 1 mL/1 kg = 51.36 mL/hour. 51.36 x 8 hours = 410.90. Rounded to the nearest whole number = 411 mL

The nurse is caring for a 12-year-old post-appendectomy client who weighs 86 pounds. The child has a temperature of 38.5ºC (101.3ºF). The nurse prepares to give the client a dose of oral acetaminophen. The order reads "Tylenol 15mg/kg/dose every 4 to 6 hours PO PRN for fever or pain." How many milligrams of Tylenol should the nurse give the client?

587 milligrams Explanation: The child's weight must first be converted to kilograms by dividing 86 by 2.2. The result is 39.1 kilograms. Next, the 39.1 kilograms must be multiplied by 15 milligrams. This answer is 587 milligrams.

A nurse is preparing to administer a prescribed dose of digoxin to an 6-month-old infant. After assessing the infant's apical pulse, the nurse decides to withhold the dose and notify the health care provider. The nurse bases this decision on which apical pulse rate?

80 beats/min Explanation: Prior to administering each dose of digoxin, the nurse would count the apical pulse for 1 full minute, noting rate, rhythm, and quality. The nurse would withhold the dose and notify the health care if the apical pulse is less than 60 in an adolescent, less than 90 in an infant.

The nurse is conducting a physical examination of an 18-month-old with suspected intussusception. Which finding would the nurse identify as the hallmark of this condition? a) Skin tenting b) A sausage-shaped mass in the upper midabdomen c) Abdominal pain and guarding d) Perianal skin tags

A sausage-shaped mass in the upper midabdomen A sausage-shaped mass in the upper midabdomen is the hallmark of intussusception. Perianal skin tags are highly suspicious of Crohn disease. Abdominal pain and guarding are also common with intussusception but are seen with many other conditions. Tenting would indicate dehydration.

The nurse is reviewing the causative organisms noted on laboratory reports. Which organism is transmitted solely by sexual contact? -Bacillus -Trichomonas -Cholera Bacterium -Borelli

Trichomonas The organism transmitted solely by sexual contact is Trichomonas. The other organisms are causes of various infections and acquired in various ways.

The nurse is caring for a pediatric client who is scheduled for the surgical removal of a Wilms tumor. Which is contraindicated in the client's care? -Intravenous fluids -Abdominal palpation -Foley catheter placement -Supine positioning

Abdominal palpation Abdominal palpation is contraindicated preoperatively in a client with a Wilms tumor. Cells may break loose and spread the tumor. Intravenous fluids and supine positioning are appropriate in the client's care. A Foley catheter is typically not placed.

A 2-year-old has a history of fever and fussiness. Which additional symptoms would make the nurse suspect a urinary tract infection? -Swollen lymph nodes -Skin rash -Increased thirst -Abdominal pain

Abdominal pain The symptoms of urinary tract infection can vary depending on the age of the child. Abdominal pain is a common symptom in children of a UTI. Swollen lymph nodes, skin rash, and thirst are not the common symptoms associated with a UTI.

The camp nurse is caring for a child who was bitten on the leg by a dangerous spider. The child is being taken to a care provider. What is the most appropriate action for the nurse to do with this child?

Apply ice to the affected area. Spider bites can cause serious illness if untreated. Bites of black widow spiders, brown recluse spiders, and scorpions demand medical attention. Applying ice to the affected area until medical care is obtained can slow absorption of the poison.

The nurse is teaching an in-service program to a group of nurses on the topic of gastrointestinal disorders. The nurses in the group make the following statements. Which statement is most accurate related to the diagnosis of gastroesophageal reflux? a) There are recurrent paroxysmal bouts of abdominal pain. b) A thickened, elongated muscle causes an obstruction at the end of the stomach. c) In this disorder the sphincter that leads into the stomach is relaxed. d) A partial or complete intestinal obstruction occurs.

In this disorder the sphincter that leads into the stomach is relaxed. Gastroesophageal reflux (GER) occurs when the sphincter in the lower portion of the esophagus, which leads into the stomach, is relaxed and allows gastric contents to be regurgitated back into the esophagus. Congenital aganglionic megacolon is characterized by persistent constipation resulting from partial or complete intestinal obstruction of mechanical origin. Colic consists of recurrent paroxysmal bouts of abdominal pain. Pyloric stenosis is characterized by hypertrophy of the circular muscle fibers of the pylorus which leads to an obstruction at the distal end of the stomach.

The nurse is educating an adolescent female at risk for hypertension. Which interventions will the nurse recommend? Select all that apply.

Increase hours of sleep. Avoid any smoking. Exercise on a daily basis. Maintain a healthy weight.

A 5-year-old boy is receiving an analgesic intravenously while in the hospital. What should the nurse do to determine whether the drug is being properly excreted from this child?

Monitor the childs fluid intake & output

Which collaborative actions will the nurse perform when caring for an infant with transposition of the great arteries scheduled for surgical repair of the defect? Select all that apply.

Provide education to the parents. Auscultate lung sounds frequently. Apply a continuous pulse oximeter. Keep oxygen saturation above 75%. Explanation: Collaborative interventions for an infant with transposition of the great arteries include providing education to parents in preparation for their infant's surgery; assessing pulse oximetry and auscultating lung sounds frequently to monitor for signs of increased pulmonary flow; and maintaining normal oxygen saturation for transposition of the great arteries at 75% to 85%. Administering indomethacin would cause closure of the ductus arteriosus, which would prevent mixing of blood.

A pediatric client is having difficulty breathing. Which nursing intervention is appropriate for this client?

Provide oxygen as needed to maintain oxygen saturation above 93% Explanation: The nurse would provide oxygen to increase oxygen saturation as needed for this child. A decrease in oxygen saturation will cause the child to have an increase in the work of breathing. The positioning does not promote an open airway. Having family members leave the room could increase the child's anxiety, which would worsen the respiratory status. Continuing to monitor the child provides no assistance or relief.

A nurse is caring for a child with tinea pedis. Which assessment finding should the nurse expect?

Red scaling rash on soles and between the toes Tinea pedis presents with red scaling rash on soles, and between the toes. Tinea capitis presents with patches of scaling in the scalp with central hair loss and the risk of kerion development (inflamed boggy mass filled with pustules). Tinea cruris presents with erythema, scaling, maceration in the inguinal creases and inner thighs.

The nurse caring for a child with a skin allergy recognizes that the highest priority in the treatment for skin allergies is aimed at:

Reducing swelling and relieving itching

Which nursing diagnosis will the nurse indicate as priority for the child following cardiac surgery for tetralogy of Fallot?

Risk for ineffective cardiopulmonary tissue perfusion Explanation: Nursing priority following cardiac surgery will focus on assessing for ineffective cardiopulmonary tissue perfusion. Monitoring for excess fluid volume, infection, and anxiety will be monitored after ensuring cardiopulmonary tissue perfusion is adequate.

When assessing an adolescent for acne, what findings would lead the nurse to identify the acne as severe? Select all that apply.

Widespread inflammatory lesions Evidence of cysts Presence of nodules Severe acne is characterized by comedones plus inflammatory lesions such as papules or pustules that are widespread and/or the presence of cysts or nodules and possibly scarring. Comedones are associated with mild acne. Papules localized to the face or back are associated with moderate acne.

The nurse is working with the mother of a toddler experiencing constipation. What information regarding childhood constipation should the nurse share with the mother? Select all that apply. a) "Reward your child for sitting on the toilet as asked, not just when they have a bowel movement." b) "Have your son sit on the toilet twice a day, after breakfast and dinner, for 5 to 15 minutes." c) "If your child has a fecal impaction, you can give him an enema." d) "Reward your child only when they have a bowel movement with a sticker." e) "You should not give your son laxatives."

• "Have your son sit on the toilet twice a day, after breakfast and dinner, for 5 to 15 minutes." • "If your child has a fecal impaction, you can give him an enema." • "Reward your child for sitting on the toilet as asked, not just when they have a bowel movement." Proper education for constipation in children includes educating the families about the importance of compliance with medication use. Many children present to their physician or nurse practitioner with fecal impaction or partial impaction. Teach parents how to disimpact their children at home; this often requires an enema or stimulation therapy. To facilitate daily bowel evacuation, the child should sit on the toilet twice a day (after breakfast and dinner) for 5 to 15 minutes. Instruct the family to keep a "star" or reward chart to encourage compliance. Parents should award the star for compliance with time sitting on the toilet and should not reserve rewards for successful bowel movements only.

What are the classic symptoms of celiacs disease

• Constipation • Diarrhea • Steatorrhea • Failure to thrive Classic symptoms of celiac disease include steatorrhea, constipation, diarrhea, failure to thrive, weight loss, abdominal distention or bloating (not a sunken abdomen), and anemia (not polycythemia).

The nurse is speaking with the mother of a child diagnosed with contact dermatitis from poison ivy. Which statement by the mother indicates a need for further education?

"As long as he takes a shower as soon as he gets inside, he shouldn't get this again."

A 4-month-old has had a fever, vomiting, and loose watery stools every few hours for 2 days. The mother calls the physician's office and asks the nurse what she should do. Which response by the nurse is most appropriate? a) "Bring the child to the office today so we can evaluate her fluid balance and determine the best treatment." b) "Continue breastfeeding as you have been doing. The fluid from the breast milk is important to maintain fluid balance." c) "Give a clear pediatric electrolyte replacement for the next few hours, then call back to report on how your child is doing." d) "Do not give the child anything to drink for 4 hours. If the fever goes down and the loose stools stop, you can resume breastfeeding."

"Bring the child to the office today so we can evaluate her fluid balance and determine the best treatment." Infants are comprised of a high percentage of fluid that can be lost very quickly when vomiting, fever, and diarrhea are all present. This infant needs to be seen by the physician based on her age and symptoms; hospitalization may be necessary for intravenous rehydration depending upon her status when assessed.

A 4-month-old has had a fever, vomiting, and loose watery stools every few hours for 2 days. The mother calls the physician's office and asks the nurse what she should do. Which response by the nurse is most appropriate? a) "Continue breastfeeding as you have been doing. The fluid from the breast milk is important to maintain fluid balance." b) "Bring the child to the office today so we can evaluate her fluid balance and determine the best treatment." c) "Do not give the child anything to drink for 4 hours. If the fever goes down and the loose stools stop, you can resume breastfeeding." d) "Give a clear pediatric electrolyte replacement for the next few hours, then call back to report on how your child is doing."

"Bring the child to the office today so we can evaluate her fluid balance and determine the best treatment." Infants are comprised of a high percentage of fluid that can be lost very quickly when vomiting, fever, and diarrhea are all present. This infant needs to be seen by the physician based on her age and symptoms; hospitalization may be necessary for intravenous rehydration depending upon her status when assessed.

The nurse is doing a presentation for a group of nursing students about the topic of menstrual disorders. After discussing the disorder secondary amenorrhea, the students make the following statements. Which statement made by the nursing students is the most accurate regarding the cause of secondary amenorrhea? -"It is caused from taking birth control pills when a girl is younger than 13 years old." -"This disorder is usually seen after a girl has had a spontaneous abortion." -"Emotional stress can be a cause of this disorder." -"This is what happens if a 16-year-old girl has never had any periods at all."

"Emotional stress can be a cause of this disorder." Secondary amenorrhea can be the result of discontinuing contraceptives, a sign of pregnancy, the result of physical or emotional stress, or a symptom of an underlying medical condition. A complete physical examination, including gynecologic screening, is necessary to help determine the cause. Primary amenorrhea occurs when a girl has had no previous menstruation. A spontaneous abortion does not cause secondary amenorrhea.

The nurse is preparing an 18-month-old for discharge after treatment for dehydration following diarrhea. What instruction would the nurse most likely include in the discharge teaching? a) "Offer her flavored gelatin if she is hungry." b) "Encourage bananas, applesauce, and crackers." c) "Give her plenty of fruit juice or soda." d) "Make sure she gets lots of clear liquids."

"Encourage bananas, applesauce, and crackers." After rehydration is achieved, it is important to encourage the child to consume a regular diet in order to maintain energy and growth. The solid foods presented are easily digested and age appropriate. The parents should avoid prolonged used of clear liquids in the child with diarrhea because "starvation stools" might result. Fluids high in glucose such as fruit juice, gelatin, and soda may worsen diarrhea. Gelatin is high in glucose and may worsen diarrhea.

The nurse is evaluating parents' understanding of atopic dermatitis. Which statement shows their understanding?

"Flare-ups of lesions are not uncommon following therapy."

A parent expresses concern about a 3-year-old child's pain while having blood drawn and asks the nurse what she can do to help the child. Which response by the nurse will be most beneficial?

"Focus on a story during the blood draw." Explanation:

The nurse is taking a health history of a 6-week-old boy with a suspected cardiovascular disorder. Which response by the mother would lead the nurse to suspect that the child is experiencing heart failure?

"He gets sweaty when he eats." Explanation: Diaphoresis with nipple feeding indicates heart failure. Shortness of breath would suggest heart failure. Children with heart failure experience subtle signs that suggest that something is wrong. Children with heart failure often display subtle signs such as difficulty feeding.

A child is being discharged from the hospital and the nurse has completed discharge teaching regarding prescribed liquid medications. Which comments by the parent demonstrates understanding of discharge instructions for safe medication administration? Select all that apply.

"I need to make sure to use the medicine dropper the pharmacy gives me instead of the syringe I use for my B12 injections." "I shouldn't use a liquid dropper from my kitchen because it may be a different measurement than one from the pharmacy." "I will be sure to not give too much of the liquid medication at one time." Only droppers given by the pharmacy for the specific medication should be used. Different syringes may have different measurements than pediatric oral syringes. Mixing medication syringes is avoided if a dropper is packaged with a certain medication since the drop size may vary from one dropper to another. Giving small amounts of liquid avoids aspiration. Pinching the child's nose increases the risk for aspiration and interferes with the development of a trusting relationship.

The nurse is caring for a child who was involved in an automobile accident in which extensive damage to the small intestine occurred. A surgical resection of the small intestine resulted in massive small intestine loss, causing short bowel syndrome. The parents voice concern to the nurse that their child will "never be the same." What is the best response by the nurse? a) "Having a chronic condition is difficult but you have to be strong for your child. You are their main support person." b) "There are so many new treatments available every day. There may be something to correct this in the near future." c) "I can't imagine what you're going through. We will be here to support and educate you on your child's condition and treatments throughout their care." d) "I know it must be difficult but there was nothing you could have done to prevent this."

"I can't imagine what you're going through. We will be here to support and educate you on your child's condition and treatments throughout their care." Being empathetic and reassuring the parents that the staff will be there to support them and educate them will provide some relief of anxiety since this is a chronic condition. Telling them their is nothing they could have done to prevent this from happening does not help the current situation. Giving hope is good, but giving possible false hope sets the family up for disappointment. Telling the family to be strong does not provide support.

The nurse has performed client teaching to a 15-year-old boy with Crohn disease, and his parents regarding the cobblestone lesions in his small intestine. Which comment by the family indicates learning occurred? a) "I may end up with a colectomy because the disease is continuous from the beginning to the end of my intestines." b) "It's unusual for someone my age to get Crohn disease." c) "I have a lot of diarrhea every day because of how my small intestine is damaged." d) "I have to be careful because I am prone to not absorbing nutrients."

"I have to be careful because I am prone to not absorbing nutrients." Crohn disease typically effects the small intestine more than the large intestine and it's onset is between the ages of 10 to 20 years. The cobblestone lesions in the small intestine prevents absorption of nutrients that normally occurs. The diarrhea is not directly related to the cobblestone lesions, and ulcerative colitis is characterized by the disease effecting the intestine(s) in a continuous pattern.

An adolescent is prescribed isotretinoin. Which statement indicates that the adolescent understands the necessary precautions associated with this drug?

"I have to make sure that I do not become pregnant while taking this drug."

The nurse is caring for a 13-year-old girl with acne vulgaris and is teaching the girl about skin care. Which response by the girl indicates a need for further teaching?

"I should avoid eating any kind of chocolate."

A topical corticosteroid is prescribed for a child with contact dermatitis. Which statement by the mother would indicate the teaching was successful?

"I should not cover the area with plastic wrap after applying the cream." An occlusive dressing such as plastic wrap over the area should not be used with topical corticosteroids. High-potency preparations should not be used. There is no need to shake topical corticosteroids. Benzoyl peroxide requires shaking before use. Applying the medication at night and rinsing off in the morning is used for coal tar preparations.

The mother of an infant tells the nurse during a routine visit to the clinic that she often notices a bulging mass in the lower abdominal and groin area when her baby cries. She asks the nurse if this is normal. How should the nurse respond? a) "I didn't notice any masses while I was assessing your infant. It may just appear they have a mass due to pressure in the abdomen when crying." b) "I will be sure to note this in your child's chart so it is something we will continue to monitor in future visits." c) "The muscle wall of infants are not yet strong so it isn't unusual to see this happening when the baby is crying or straining." d) "I understand your concern. I will be sure to document this in your child's medical record and report this information to your child's physician immediately."

"I understand your concern. I will be sure to document this in your child's medical record and report this information to your child's physician immediately." The mother is describing common symptoms of an inguinal hernia. It may be possible to visualize the mass, but often the mass is seen only during crying or straining, making it difficult to actually identify in the clinic setting. It is important to notify the physician so treatment can be initiated. (less)

The nurse is caring for an infant recently diagnoses with thrush and was prescribed nystatin. Which statement by the infant's mother would suggest a need for further education? a) "I will add the nystatin to her bottle four times per day." b) "I will use a cotton tipped applicator to apply the medication to her mouth." c) "I will make sure to clean all of her toys before I give them to her." d) "I will watch for diaper rash."

"I will add the nystatin to her bottle four times per day." Administer nystatin suspension four times per day following feeding, not mixed in the bottle, to allow the medication to remain in contact with the lesions. In the younger infant, apply nystatin to the lesions with a cotton-tipped applicator. Infants and young children often mouth their toys, so it is important to clean them appropriately. Explain to parents of infants with thrush the importance of reporting diaper rash because fungal infections in the diaper area often occur concomitantly with thrush and also need to be treated.

The nurse is caring for an infant recently diagnoses with thrush and was prescribed nystatin. Which statement by the infant's mother would suggest a need for further education? a) "I will make sure to clean all of her toys before I give them to her." b) "I will use a cotton tipped applicator to apply the medication to her mouth." c) "I will watch for diaper rash." d) "I will add the nystatin to her bottle four times per day."

"I will add the nystatin to her bottle four times per day." Administer nystatin suspension four times per day following feeding, not mixed in the bottle, to allow the medication to remain in contact with the lesions. In the younger infant, apply nystatin to the lesions with a cotton-tipped applicator. Infants and young children often mouth their toys, so it is important to clean them appropriately. Explain to parents of infants with thrush the importance of reporting diaper rash because fungal infections in the diaper area often occur concomitantly with thrush and also need to be treated.

The nurse is caring for a child with an order for PO prednisone. Which statement by the child's mother would indicate a need for further education?

"I will give it to her at least 1 hour before all of her meals."

The student nurse is caring for a child with the nursing diagnosis "Risk for fluid volume deficit related to inadequate oral intake". Which statement by the student would indicate a need for further education by the nursing instructor? a) "I will teach her mother to give her small drinks frequently." b) "I will weigh her every morning at the same time." c) "I will monitor her IV line to help maintain her fluid volume." d) "I will make sure there is plenty of orange juice available. It's her favorite juice."

"I will make sure there is plenty of orange juice available. It's her favorite juice." Proper interventions for children at risk for fluid volume deficit include maintaining IV line and administer IV fluid as ordered to maintain fluid volume. Offer small amounts of oral rehydration solution frequently to maintain fluid volume. Small amounts are usually well tolerated by children with diarrhea and vomiting. Avoid high-carbohydrate fluids such as Kool-Aid and fruit juice, as they are low in electrolytes, and increased simple carbohydrate consumption can decrease stool transit time. Daily weights are one of the best indicators of fluid volume status in children.

The nurse is providing education to a teenaged boy diagnosed with impetigo. Which statement by the boy indicates the need for further education?

"I will need to cover my son's skin lesions with bandages until it has healed." Impetigo is an infectious bacterial infection. The crusts should be removed after soaking prior to applying topical medications. Leaving the lesions open to air is not contraindicated. Children diagnosed with impetigo may attend school during treatment.

A nurse has just given otic medication instructions to the parents of a 12-year-old child. Which statement would indicate that the parents need further education concerning the medication?

"I will pull the outer ear down and back before administering the medication." Explanation: The proper technique to instill ear drops in a child older than age 3 involves pulling the pinna up and back. Otic medication should not be administered if it is cold. Cold medication may cause discomfort and produce vomiting or vertigo in the child. If an otic medication must be refrigerated, it should be warmed to room temperature in the palms of the hands. Proper otic administration technique involves holding the dropper 0.5 in (1.25 cm) above the ear canal and being careful not to touch the dropper to the ear to prevent contamination of the dropper with microorganisms. For children young than 3, the parent would pull the pinna down and back.

The nurse is caring for an infant with diaper dermatitis. Which statement by the child's parent would indicate a need for further education?

"I will use rubber pants over the cloth diapers in the future." Prevention and management of diaper dermatitis includes avoiding rubber pants, avoiding diaper wipes with fragrance or preservatives. Treatment of a rash includes allowing the child to go diaperless for a period of time each day and using a warm blow dryer on the area for 3 to 5 minutes.

The primary health care provider prescribed ketoconazole for a child with ringworm. Which statement by the parents indicates the nurse needs to provide additional teaching on the prescription?

"I will wrap the skin tightly after applying the medication." Explanation: Ketoconazole is an antifungal used to treat tinea infections. The nurse would teach to avoid covering treated skin areas with tightly. The area needs to allow for air to circulate to the skin in order to limit side effects. All other statements indicate correct understanding.

The nurse has provided teaching of nonpharmacologic pain management to the parents of a 3-year-old child experiencing postoperative pain. Which comments by the parents indicate that the teaching was effective? Select all that apply.

"I'm not sure if I am imagining it, but I think my child seems to be in less pain when I rock her." "My child seems to cry a lot less with medical procedures if we are reading a book together." "One of the nurses blows bubbles with our child every time they are preparing to perform a procedure, then allows our child to do the same during the procedure. It really helps." "I try to remind our child to think about our dog at home. Our dog is like a big cuddle toy to our child."

The nurse has completed client teaching with a 16-year-old female who has been prescribed Accutane (isotretinoin) for cystic acne. Which statements indicate learning has occurred? Select all that apply.

"If I am sexually active I need to let my doctor know." "This is not a drug to be used for all forms of acne. My sister has minor acne so I told her this wasn't for her." "It's important I get my CBC blood test when my doctor orders it."

An infant with poor feeding is suspected of having a congenital heart defect. The parents are asking why a chest x-ray is necessary in their infant. What is the best response from the nurse?

"It will determine if the heart is enlarged." Explanation: Chest x-rays= are performed to see if the heart is enlarged. This will determine if the heart muscle is increasing in size. EKG= Disturbances in heart conduction are detected by an EKG. Echocardiogram= Visualizing where blood is being shunted is through the echocardiogram. MRI= The image used to clarify the structures of the heart is the MRI.

The parents of a 5-year-old child with a cardiovascular disorder tell the nurse they don't understand why their child isn't gaining weight, "We make sure our child has 3 very nutritious meals every day." How should the nurse respond?

"It's great you are providing nutritious meals, but smaller frequent meals will tire your child less and promote weight gain." Explanation: Small, frequent feedings will reduce the amount of energy required to feed or eat and prevents overtiring the child. Questioning if the parents are making nutritious foods or foods the child likes does not address the issue of energy expenditure when eating 3 larger meals per day. Just stating that it is "hard to get your child to eat enough" also does not address the situation.

The mother of a young child, who has been treated for a bacterial urinary tract infection, tells the nurse her daughter has a white thick covering over her tongue. The mother states that she "has tried everything to get it off my child's tongue." How should the nurse respond? a) "That is a common side effect after taking an antibiotic. It will go away after the antibiotic is out of the system." b) "Have you tried using a toothbrush to get it off?" c) "It is likely an infection was caused by the antibiotic for the urinary tract infection. I am sure a different antibiotic will help it." d) "It's not unusual for a fungal infection to occur while taking an antibiotic for an infection. I will let your primary care provider know so we can get it treated."

"It's not unusual for a fungal infection to occur while taking an antibiotic for an infection. I will let your primary care provider know so we can get it treated." Thrush is a fungal infection that can occur on the tongue while on an antibiotic for an unrelated bacterial infection. The antibiotic destroys normal flora, which allows the fungal infection to occur. Thrush requires an antifungal agent, such as nystatin liquid, to destroy the infection. Additional antibiotics will not help since it is a fungal infection.

The nurse is preparing a 7-year-old girl for discharge after treatment for nephrotic syndrome. Which instructions would the nurse include in the discharge teaching plan for the parents? -"Let's meet with the dietitian and plan some meals." -"She must severely restrict her sodium intake." -"She should try to avoid protein." -"Here is some written information from the dietitian."

"Let's meet with the dietitian and plan some meals." Consultation with a dietitian would be most helpful for meal planning because so many of children's favorite foods are high in sodium. Restricting sodium may not be necessary if the child is not edematous; in addition, the statement does not teach. Protein-rich snacks should be encouraged. The nurse needs to provide the parents with specific instructions, assistance, and resources in addition to simple written instructions.

The nurse is caring for a child who is experiencing postoperative pain after having undergone surgery several hours ago. The child's parent reports having taken meperidine for postoperative pain and wonders if that medication would be of benefit to the child. What response by the nurse is indicated?

"Meperidine is associated with toxicity issues in children and is usually avoided."

A child will be undergoing a Holter monitor test. Which statement by the mother indicates the need for further instruction?

"My child cannot have any thing to eat or drink after midnight the day of the test." Explanation: Ambulatory electrocardiographic monitoring (Holter) testing is an exam that spans approximately 24 hours. The test is done to review the activity of the heart. The individual is encouraged to follow their normal activities during the test. There is no need for the child to be NPO prior to or during the test.

The nurse is caring for a 4-year-old with oral vesicles and ulcers from herpangina. The child is refusing fluids due to the pain and the mother is concerned about his hydration status. Which of the suggestions would be most appropriate? a) "Try some Anbesol or Kank-A." b) "Encourage him to have some soda." c) "Offer 'magic mouthwash' followed by a popsicle." d) "Offer him some orange juice."

"Offer 'magic mouthwash' followed by a popsicle." Children are more likely to cooperate with interventions if play is involved. "Magic" analgesic mouthwash followed by a popsicle is most likely to alleviate some pain and then provide hydration. Soda should be avoided because it can cause stinging and burning. Orange juice should be avoided because it can cause stinging and burning. Anbesol might be helpful but it will likely be difficult to apply. Additionally, oral analgesics are often necessary.

The nurse is caring for the parents of a newborn who has an undescended testicle. Which comment by the parents indicates understanding of the condition? -"Our son may need surgery on his testes before we are discharged to go home." -"Our son may have to go through life without two testes." -"Our son's condition may resolve on its own." -"Our son will likely have a high risk of cancer in his teen years as a result of this condition."

"Our son's condition may resolve on its own." Normally both testes will descend prior to birth. In the event this does not happen the child will be observed for the first 6 months of life. If the testicle descends without intervention further treatment will not be needed. Surgical intervention is not needed until after 6 months if the testicle has not descended.

The nurse is preparing a 6-year-old for a venipuncture. The boy appears anxious and is crying. How can the nurse foster feelings of control to help minimize his anxiety about the procedure?

"Pick your favorite Band-Aid and show me which arm to use." Allowing the child options related to the style of the Band-Aid and the extremity to use gives the child some control over the happenings. Offering a pinwheel is a distraction technique. Encouraging the parent to hold the child during the procedure promotes feelings of security. Encouraging the child or parents to ask questions facilitates communication.

A parent asks the nurse, "What is precocious puberty?" The nurse's response should be based on which statement? -"Precocious puberty is when children are going through puberty." -"Precocious puberty is early sexual development." -"Precocious puberty only occurs in boys, not girls." -"Precocious puberty is when girls experience a heavy period."

"Precocious puberty is early sexual development." Precocious puberty is the early sexual development or maturation of a girl or boy. It occurs most often in girls, not boys, and does not relate to a heavy menses.

The nurse is caring for an infant. The infant's mother asks the mother, "what did the doctor mean when he said she may have regurgitation?" What response by the nurse is appropriate? a) "Regurgitation is the backflow of stomach contents up into the esophagus or mouth." b) "Regurgitation is not normal in infants. She will need more testing to see what is causing this." c) "Regurgitation is just another term for vomiting. All infants vomit some." d) "Regurgitation is when an infant can't tolerate their formula. You will need to switch."

"Regurgitation is the backflow of stomach contents up into the esophagus or mouth." Regurgitation is the backflow of stomach contents up into the esophagus and/or oral cavity. The muscle tone of the lower esophageal sphincter is not fully developed until age 1 month, so infants younger than 1 month of age frequently regurgitate after feedings. Many children younger than 1 year of age continue to regurgitate for several months, but this usually disappears with age. (

The nurse is collecting data on a 2 ½-year-old child admitted with a diagnosis of gastroenteritis. When interviewing the caregivers, which question is most important for the nurse to ask? a) "What foods has your child eaten during the last few days." b) "Tell me about the types of stools you child has been having." c) "How long has your child been toilet trained?" d) "How many times a day does your child urinate?"

"Tell me about the types of stools you child has been having." For the child with gastroenteritis, the interview with the family caregiver must include specific information about the history of bowel patterns and the onset of diarrheal stools, with details on number and type of stools per day. Recent eating patterns, if the child is toilet trained, and how many times a day the child urinates are important questions, but the highest priority is gathering data regarding the stools and stool pattern.

A nurse is examining a 10-year-old girl who has a heart murmur. On auscultation, the nurse finds that the murmur occurs only during systole, is short, and sounds soft and musical. When she has the girl stand, she can no longer hear the murmur on auscultation. Which statement should the nurse make to the girl's mother in response to these findings?

"Your daughter has an innocent heart murmur, which is nothing to worry about." Explanation: The symptoms described indicate an innocent heart murmur. Although innocent murmurs are of no consequence, parents need to be told when their child has one because this finding will undoubtedly be discovered again at a future health assessment or during a febrile illness, anxiety, or pregnancy. Activities need not be restricted when a child has an innocent murmur and the child requires no more frequent health appraisals than other children. If a murmur is present as the result of heart disease or a congenital disorder, it is an organic heart murmur. Reference:

The caregiver of a 1-year-old boy calls the nurse, upset that his wife has just told him that their son is being given a hormone. His wife says that the pediatrician called it human chorionic gonadotropic hormone but that is all she understood. The nurse most accurately clarifies the caregiver's question by making which statement regarding the son's treatment? -"Without the hormone your son will have fluid that will collect in his scrotum." -"Without the treatment your child's gonads will not reach normal size." -"The doctor is hoping that the hormone will cause your son's undropped testes to move into their proper place." -"Your child's testes have not dropped, so the hormone is being administered to avoid causing degeneration until they do."

"The doctor is hoping that the hormone will cause your son's undropped testes to move into their proper place." Shortly before or soon after birth, the male gonads (testes) descend from the abdominal cavity into their normal position in the scrotum. Occasionally one or both of the testes do not descend, which is a condition called cryptorchidism. The testes are usually normal in size; the cause for failure to descend is not clearly understood. A surgical procedure called orchiopexy is used to bring the testes down into the scrotum and anchor them there. Some physicians prefer to try medical treatment such as injections of human chorionic gonadotropic hormone before doing surgery. If this is unsuccessful in bringing down the testes, orchiopexy is performed. If both testes remain undescended, the male will be sterile. If the processus does not close, fluid from the peritoneal cavity passes through, causing hydrocele. If the hydrocele remains by the end of the first year, corrective surgery is performed.

A child who is receiving TPN has developed the need to have insulin injections. The child's mother questions this and states that her child does not have diabetes. What is the appropriate response by the nurse?

"The feedings are high in sugar and insulin is needed to manage this." Explanation: Glucose levels may be elevated when TPN is administered. While illness can impact serum glucose levels, this is not an appropriate response. Telling the parent there is no need to worry minimizes concerns and is not a correct response. The child does not have diabetes but warrants insulin coverage.

A physician recommends a gastrostomy for a 4-year-old client with an obstruction. The parents ask the certified wound, ostomy, and continence nurse (CWOCN) what the surgery entails. What is the nurse's best response? a) "The surgery will create an opening to the large intestine." b) "The surgery will create an opening to the small intestine." c) "The surgery is performed to create an opening between the esophagus and the neck." d) "The surgery creates an opening between the stomach and abdominal wall."

"The surgery creates an opening between the stomach and abdominal wall." Ostomies can be created at various sites in the GI tract, depending on the child's clinical condition. A gastrostomy provides an opening between the stomach and the abdominal wall, and an esophagostomy communicates between the esophagus and an external site on the neck. Ostomies may be created at various sites in the small intestine (e.g., jejunostomy, ileostomy) or in the large intestine (e.g., colostomy).

The nurse is discussing the treatment of congenital aganglionic megacolon with the caregivers of a child diagnosed with this disorder. Which statement is the best explanation of the treatment for this diagnosis? a) "Your child will be treated with oral iron preparations to correct the anemia." b) "We will give enemas until clear and then teach you how to do these at home." c) "Your child will receive counseling so the underlying concerns will be addressed." d) "The treatment for the disorder will be a surgical procedure."

"The treatment for the disorder will be a surgical procedure." Treatment of congenital aganglionic megacolon involves surgery with the ultimate resection of the aganglionic portion of the bowel. Chronic anemia may be present, but iron will not correct the disorder. Enemas may be given to initially achieve bowel elimination, but they will not treat the disorder. Differentiation must be made between this condition and psychogenic megacolon because of coercive toileting or other emotional problems. The child with aganglionic megacolon does not withhold stools or defecate in inappropriate places, and no soiling occurs.

The nurse is caring for a 10-year-old child experiencing nocturnal enuresis with no physiologic cause. The child states, "I am embarrassed and I wish I could stop this right now!" How will the nurse respond? -"You will grow out of this eventually; you just need to be patient." -"There are several things we can do to help you achieve this goal." -"You are not alone. There are almost 5 million people that have enuresis." -"You can wear pull-ups to bed and, since they look like underwear, no one will know."

"There are several things we can do to help you achieve this goal." The child wants to stop this problem immediately, so the nurse's most therapeutic response is to assure the child that enuresis is indeed solvable. For some children, learning about the high prevalence of the problem may provide consolation. However, this may not alleviate the child's embarrassment and it does not address the desire for solutions. Telling the child that he or she will "grow out of this" downplays the embarrassment and does not address the desire to solve the problem. Pull-ups conceal the consequences of enuresis but do not provide a solution.

The nurse is providing family education for the administration of ibuprofen. Which response indicates a need for further teaching?

"This can be taken with other medications we have at home that didn't require a prescription." The nurse must emphasize that the parents should carefully read labels of over-the-counter medications they already have or will purchase. Some may contain ibuprofen or other nonsteroidal anti-inflammatory drugs, and if given in conjunction with ibuprofen may lead to overdose. The other statements are correct.

The nurse is caring for a 17-year-old child who was sprained her ankle. The physician has prescribed ibuprofen to manage the pain. What statement by the teen indicates the need for further instruction?

"This medication should be taken on an empty stomach."

A child has an order for an erythrocyte sedimentation rate (ESR). The child's mother asks what the purpose of the test is. What is the best response by the nurse?

"This test will tell if your child has an infection or inflammation somewhere in their body."

The nurse is preparing to administer an intramuscular immunization to a 5-year-old child. What statement to the child is appropriate for inclusion in the preadministration period?

"This will help prevent you from getting sick." Explanation: When providing teaching to a child it is important to be open, honest and provide developmentally appropriate information. Explaining that this will prevent later illness is something a child can understand. Saying that pain may result if movement occurs is a scare tactic and counterproductive. Yes, this is an immunization but this is terminology that is too complex for a child. Using the word "shot" is scary for the child and should not avoided if possible.

The nurse is caring for a newborn diagnosed with imperforate anus following delivery. The physician has discussed the treatment options and prognosis with the parents. The nurse is talking with the parents and determines that learning has occurred when the parents make which statement? a) "Since our baby has a defect high in the anorectal opening there is a good chance that stool continence won't be a problem." b) "We are worried that our child may have other congenital problems that we aren't aware of." c) "We aren't sure if our baby will need surgery at some point for this problem." d) "We know we will need to use baby wipes around the anal area after surgery to prevent infection."

"We are worried that our child may have other congenital problems that we aren't aware of." Imperforate anus is a congenital malformation of the anorectal opening. Other congenital anomalies may be associated with imperforate anus in 50% of cases. Surgical intervention is needed for both high and low types of imperforate anus. After repair, only about 30% with a high defect will achieve continence. To decrease the drying associated with frequent cleaning, avoid baby wipes and frequent use of soap and water.

The nurse is educating the parents of a 5-month-old on how to administer an oral antibiotic. Which response indicates a need for further teaching?

"We can mix the antibiotics into his formula or food." Explanation: Never mix a medication with formula or food. The child may associate the bitter taste with the food and later refuse to eat it.

A nurse providing teaching on ways to promote skin hydration for the parents of an infant with atopic dermatitis. Which response indicates a need for further teaching?

"We should bathe our child in hot water, twice a day."

The nurse is caring for a 9-year-old boy with episodes of chronic pain. The nurse is educating the parents how to help the child manage pain nonpharmacologically. Which statement indicates a need for further teaching?

"We should start the method after he feels pain."

The nurse is caring for a 7-year-old who is being discharged following surgery with a Gore® Helex device to repair an atrial septal defect. The parents of the child demonstrate understanding of the procedure with which statements? Select all that apply.

"We will be sure to not allow our child to ride a bicycle for at least 2 weeks." "We will be sure to monitor our child for any signs of infection and notify the doctor if we notice any." "We know how important our child's medications are so we will write out a schedule to be sure medications are taken as prescribed."

The nurse is caring for an infant with Down syndrome who has been diagnosed with atrioventricular canal defect. The parents ask the nurse, "Why do you not put oxygen on our baby more often when his lips and fingernails are blue?" What is the best response by the nurse?

"While it seems that oxygen would help, it actually worsens this condition. Treating the cause of the disease will help." Explanation: For children with congenital defects with increased pulmonary blood flow, oxygen supplementation is not helpful. Oxygen acts as a pulmonary vasodilator. If pulmonary dilation occurs, pulmonary blood flow is even greater, causing tachypnea, increasing lung fluid retention, and eventually causing a much greater problem with oxygenation; therefore, preventing the development of pulmonary disease via early surgical correction is essential. Although oxygen must be ordered by the physician and it isn't the best treatment, the nurse stating, "While it seems that oxygen would help, it actually makes the condition worse. Treating the cause of the disease will help" best answers the question while also showing empathy.

The nurse is caring for a group of children who have had recent surgery. Which children will the nurse question to determine the location of their pain? Select all that apply.

3-year-old post tonsillectomy 4-year-old with a fractured tibia 5-year-old with sickle cell crisis 6-year-old with juvenile arthritis 7 year old post appendectomy DONT ASK THE 2 YEAR OLD WHO DOESNT UNDERSTAND WELL ENOUGH

The nurse is caring for a child with suspected child abuse-induced burns. Which assessment findings would support this?

A burn to the entire right hand up to 2 cm above wrist with consistent edges A stocking/glove pattern on the hands or feet (circumferential ring appearing around the extremity, resulting from a caregiver forcefully holding the child under extremely hot water) is one sign is one sign of child abuse-induced burns.

The nurse is caring for a child who receives dialysis via an AV fistula. Which finding indicates an immediate need to notify the physician? -Presence of a bruit -Presence of a thrill -Dialysate without fibrin or cloudiness -Absence of a thrill

Absence of a thrill The nurse should always auscultate the site for presence of a bruit and palpate for presence of a thrill. The nurse should immediately notify the physician if there is an absence of a thrill. Dialysate without fibrin or cloudiness is normal and is used with peritoneal dialysis, not hemodialysis.

The dermatologist treating a 16-year-old girl with a history of severe acne has ordered a pregnancy test so she can be started on a course of isotretinoin. The teen's caregiver has said that her daughter is a virgin and she refuses to allow her to have the required pregnancy testing. What would be the best action for the nurse to take?

Acknowledge the caregiver's discomfort about the pregnancy testing but encourage her to allow the daughter to have the testing so that she can use the medication.

Which nursing diagnosis would best apply to a child with rheumatic fever?

Activity intolerance related to inability of heart to sustain extra workload Explanation: Acute rheumatic fever affects the joints, central nervous system, skin, and soft tissue. It causes chronic, progressive damage to the heart and valves. Children with rheumatic fever need to reduce activity to relieve stress on the heart and joints during the course of the illness. Rheumatic fever does not produce cardiomegaly nor does it interfere with respirations or the ability to oxygenate the body. Children with rheumatic fever may develop chorea. These movements are involuntary and are not related to hyperexcitability.

A 7-year-old boy tells the nurse that his head sometimes hurts after he eats ice cream. The nurse recognizes that this type of pain is:

Acute referred pain Acute pain means sharp pain, as is the case in this scenario. It generally occurs abruptly after an injury. The pain of a pin prink is an example. Chronic pain is pain that lasts for a prolonged period or beyond the time span anticipated for healing. Referred pain is pain that is perceived at a site distant from its point of origin. In this case, the typical ice cream "brain freeze" is a headache that results from the contact of the cold ice cream with the digestive tract.

A mother brings her 3-year-old son to the ER and tells the triage nurse that he has been vomiting blood. A medical history determines that the 3-year-old has no history of GI disturbances and his only symptoms are a slightly elevated fever and vomiting bright red blood. Based on these symptoms, what condition might the nurse suspect? a) Intussusception b) Acute upper GI bleeding c) GI tract obstruction d) Gastroesophageal reflux

Acute upper GI bleeding Vomiting bright red blood indicates acute upper GI bleeding. Emesis containing brown, foul-smelling stool indicates GI obstruction. Stool with red blood and mucus is associated with intussusception. Bleeding is not generally indicative of gastroesophageal reflux.

A 3-year-old child with asthma and a respiratory tract infection is prescribed an antibiotic and a bronchodilator. The nurse notes the following during assessment: oral temperature 100.2°F (37.9°C), respirations 52 breaths/minute, heart rate 90 beats/minute, O2 saturation 95% on room air. Which action will the nurse take first?

Administer the bronchodilator via a nebulizer. Explanation: The nurse would first administer the bronchodilator to open the child's airway and facilitate breathing. Once the airway was open, the nurse could administer oxygen, if indicated. At this time, the child's saturation level is normal but it should be monitored. The nurse would then administer the antibiotic medication. The heart rate is within normal range for a child of this age (65 to 110 beats/minute); therefore, a cardiac monitor is not needed at this time.

A neonate is to receive a hepatitis B vaccine within a few hours after birth. What is the best approach for the nurse to take when giving this medication?

Administer the medication in the neonate's vastus lateralis with a 25-gauge needle. Explanation: The vastus lateralis site is a safe choice for intramuscular (IM) injections in a neonate. A 22- to 25-gauge needle is recommended for neonates, but the nurse must assess the neonate's size before determining needle size to use. The 25-gauge needle is recommended for neonates. The dorsogluteal site should not be used until school age. Neither the deltoid muscle nor the dorsogluteal muscle are recommended IM sites for neonates. These muscles should not be used until toddler age or older. The volume of the medication should not exceed 0.5 ml per injection until the child is preschool age.

A family the nurse is working with administers cycled total parenteral nutrition (TPN) over a 12-hour period at night to free their teenage son for activities during the day. In teaching this family, what areas would the nurse stress? Select all that apply.

Administering the solution at half-rate during the first and last hour of the infusion Inspecting the insertion site of the catheter regularly

Visceral Explanation: Visceral pain is often produced by disease. It usually is diffuse and poorly localized and is described as a deep ache or sharp stabbing sensation that may be referred to other areas. Deep somatic pain typically involves the muscles, tendons, joints, fasciae, and bones and causes dull, aching, or cramping pain. Neuropathic pain usually results in burning, tingling, shooting, squeezing, or spasm-like pain. Chronic pain is defined as pain that continues past the expected point of healing for injured tissue.

After achieving a relaxed state, begin by guiding the 13-year-old client to image of walking down a sandy beach and collecting seashells, a favorite activity. Imagery begins with achieving a relaxed state. Then, the nurse guides the child to choose a favorite place. Imagery involves the use of imagination to create a mental picture. This image is positive and pleasurable. The child associates the image with colors, sounds, smells, or feelings. When using guided imagery, the nurse should not lead the child. The nurse lets the child become immersed in the personal image and take command of the experience. Guided imagery is not appropriate for preschool-age children and toddlers.

A 4-month-old infant is experiencing dermatitis in the diaper area. What treatments will be beneficial to this condition? Select all that apply.

Allow the diaper area to air dry. Apply petroleum jelly to the diaper area. Apply ointment with vitamin A to the diaper area. Use ointments containing zinc on the diaper area.

The nurse is preparing to remove an IV device from the arm of a 6-year-old girl. Which approach is best for minimizing fear and anxiety?

Ask the child if they would like to help remove the tape from the IV. The nurse needs to openly discuss the procedure with the child at an age appropriate level. The nurse should explain what is to occur and enlist the child's help in the removal of the tape or dressing. This provides the child with a sense of control over the situation and also encourages his or her cooperation. The nurse should avoid using scissors to remove the tape or dressing and the comment regarding cutting may be perceived as threatening and/or frightening. The procedure may be minorly uncomfortable so it is best to be honest with the child.

The nurse is caring for a 13-year-old client. The nurse prepares and verifies several medications and brings them and the medication administration record to the client's room. The nurse observes that the client is not wearing an identification band. Which action will the nurse to take?

Ask the client to recall his or her name and date of birth. Explanation: If the client does not have an identification band in place, the nurse must first identify the client before administering any medication. A parent should identify an infant or younger child. The nurse can ask an older child his or her name and date of birth or other identifier. There is no need to notify the prescribing health care provider. The nurse should call the admitting department at a later time to obtain a new identification band. Locating another RN to identify the client is not necessary.

The nurse observes the interactions of parents with their infant who was born with a cleft lip. The mother is attempting to feed the baby, but does not make eye contact. The father is watching television with his back turned to the mother and baby. What psychosocial nursing intervention would be most helpful to this family? a) Explain to the parents that surgical intervention will fix the defect in the baby's lip. b) Teach the mother the appropriate technique for breast-feeding an infant with cleft lip. c) Refer the family to a social worker or mental health practitioner. d) Ask the parents if they have any questions regarding the care of their child.

Ask the parents if they have any questions regarding the care of their child. The family's emotional response to the birth of a child with cleft lip, palate, or both may range from grief to anger to denial. The nurse should encourage the parents to express their feelings and provide the parents with opportunities and support for normal infant-parent interactions

A child is diagnosed with rheumatic fever. For which medication will the nurse educate the caregivers?

Aspirin Explanation: Medications used in the treatment of rheumatic fever include penicillin, salicylates (aspirin), and corticosteroids. Insulin would be given for diabetes and dilantin for seizure disorders. Antivirals are not relevant to the treatment of a bacterial infection.

Coarctation of the aorta demonstrates few symptoms in newborns. What is an important assessment to make on all newborns to help reveal this condition?

Assessing for the presence of femoral pulses **As a result, the blood pressure will be higher in the upper part of the body and decreased in the lower part of the body.** Explanation: Coarctation of the aorta is a defect where there is narrowing of the aorta, which is the largest vessel carrying oxygenated blood to the body. As a result of this narrowing, blood flow is impeded causing pressure to increase in the area proximal to the defect and decrease distal to the defect. As a result, the blood pressure will be higher in the upper part of the body and decreased in the lower part of the body. The pulses will be full or bounding in the upper part of the body and weak or absent in the lower part of the body. When assessing an infant with coarctation there may be weak or absent femoral pulses. There will also be differences in the blood pressure readings. These infants may or may not have a murmur and will be no more fussy than other babies.

A 5-year-old is being prepared for diagnostic cardiac catheterization, in which the catheter will be inserted in the right femoral vein. What intervention should the nurse take to prevent infection?

Avoid drawing a blood specimen from the right femoral vein before the procedure Explanation: Because the vessel site chosen for catheterization must not be infected at the time of catheterization (or obscured by a hematoma), never draw blood specimens from the projected catheterization entry site before the procedure (generally a femoral vein). The other interventions listed are performed for reasons other than prevention of infection. Children scheduled for the procedure are usually kept NPO for 2 to 4 hours beforehand to reduce the danger of vomiting and aspiration during the procedure. Be certain to record pedal pulses for a baseline assessment. The site for catheter insertion is locally anesthetized with EMLA cream or intradermal lidocaine.

What information would be included in the care plan of an infant in heart failure

Begin formulas with increased calories. Infants with heart failure need increased calories for growth. The infants are typically given smaller, more frequent feedings to decrease the amount given and to help conserve energy for feeding. They often are given a higher-calorie formula. The infant should be placed in an upright position or in a car seat to increase oxygenation. The infant should not have any pressure on the diaphragm while in this position. Vomiting is a sign of digoxin toxicity and this should be considered before administering.

The nurse is collecting data on a 6-year-old child admitted with acute glomerulonephritis. Which vital sign would the nurse anticipate with this child's diagnosis? -Pulse rate 112 bpm -Pulse oximetry 93% on room air -Respirations 24 per minute -Blood pressure 136/84

Blood pressure 136/84 Hypertension appears in 60% to 70% of clients during the first 4 or 5 days with a diagnosis of acute glomerulonephritis. The pulse of 112 would be a little high for a child this age, but not a concern with this diagnosis. The other vital signs are within normal limits for this age child.

A client comes to the clinic following an ankle injury. The nurse anticipates which therapeutic effect of heat if applied to injury site?

Capillaries dilate and edema reduces to the lower extremity

The nurse is caring for a client brought to a pediatric clinic for swelling in the lower extremities with reddened skin that has undefined borders and pits slightly when pressed. What is the most likely diagnosis of the client's skin alteration?

Cellulitis

A nurse is administering an enteral feeding to a child with a G-tube. What is a recommended step in this procedure? a) After feeding, flush the tube with a small amount of saline and leave the gastrostomy tube open for 2 to 5 minutes. b) Position with the head of the bed lowered at a 20° angle. c) Check for gastric residual before starting feeding by gently aspirating from the tube with a syringe. d) Administer feeding by connecting the syringe barrel to the tube and pouring formula into the syringe with a syringe plunger.

Check for gastric residual before starting feeding by gently aspirating from the tube with a syringe. The nurse should check for gastric residual before starting feeding by gently aspirating from the tube with a syringe or positioning the tube below the level of the stomach with only the barrel of the syringe attached. The client should be positioned with his or her head elevated 30° to 45° and formula should be allowed to flow with gravity, not plunged unless it is clogged. After feeding, the nurse should flush the tube with a small amount of water, unless contraindicated, and leave the gastrostomy tube open for 5 to 10 minutes after feeding to allow for escape of air.

The nurse is preparing to administer regular insulin to a nonverbal pediatric client. Which action will the nurse perform prior to administering the medication?

Check the full name and birth date on the client's wristband with the medication administration record. Explanation: When administering medications to a child, the nurse needs to use at minimum two client identifiers that are directly associated with the client and the medication to be given, such as full name, client ID number, and birth date. The nurse will take the medication administration record to the room to perform a "double-identifier" check. A client's identity must be verified with two acceptable identifiers, not just one. There is no need to have another nurse verify or have the parent state the client's information. A room number or a bed number is not an acceptable identifier.

A 6-year-old client is prescribed to receive an oral antibiotic. What should the nurse do before giving the child this medication?

Check to see if the child can swallow pills. Many children do not have enough coordination to swallow tablets or pills until they are 6 or 7 years of age. Children younger than 9 years of age often have difficulty swallowing tablets. This can make getting a child to agree to try an oral medication difficult. The nurse needs to check to see if the child can swallow pills before providing the oral medication. Drinking a glass of water before giving the medication will not determine if the child can swallow an oral medication. Giving the oral medication at the time of the next meal does not necessarily mean that the child will be able to swallow the oral medication. The nurse should not threaten to give the medication with an injection.

The nurse is reviewing the laboratory test results of several children who have come to the clinic for evaluation. Which child would the nurse identify as having the least risk for hyperlipidemia?

Child A with a total cholesterol of 150 mg/dl and low-density lipoprotein (LDL) of 80 mg/dl. Explanation: ACCEPTABLE RANGE= Total cholesterol levels below 170 mg/dl and LDL levels less than 100 mg/dl are considered within the acceptable range. BORDERLINE= Total cholesterol levels between 170 and 199 mg/dl and LDL levels between 100 and 129 mg/dl are considered borderline. GREAT RISK= Total cholesterol levels greater than or equal to 200 mg/dl and LDL levels greater than or equal to 130 mg/dl are considered elevated and place this child at greatest risk.

The nurse is assessing a child who has had an open wound for the last 2 weeks with no signs of healing. The parents report they have been using over-the-counter antibiotic ointment with no signs of improvement. The child is active in football in the fall and the swim team in the winter. What disorder does the nurse suspect?

Community acquired MRSA Risk factors for community acquired MRSA (CA-MRSA) are turf burns, towel sharing, participation in team sports, or attendance at day care or outdoor camps. The condition usually presents with a moderate to severe skin infection or with an infection that is not responding as expected to therapy

The nurse is assessing the heart rate of a 6-month-old infant and determines it to be 82 beats/min. What action should the nurse take first?

Conduct a focused cardiovascular assessment. Explanation: The normal infant heart rate averages 90 to 160 beats/min. This infant's heart rate is low. The nurse should first conduct a focused cardiovascular assessment. After that is completed, the findings can be reported to the health care provider. Obtaining a health history can be beneficial, but with a low heart rate the nurse would want to know first if there are any other clinical signs indicating a problem or a reason for the heart rate. The heart rate should be reassessed, but not necessarily in a 5-minute window. The heart rate should be assessed via apical pulse for a full 60 seconds.

A 4-year-old child is scheduled for an MRI. The child's parent is informed that the child will be free of pain but sedated to ensure stillness during the procedure. Which type of anesthesia does the nurse expect this child to have?

Conscious sedation Explanation: Conscious sedation refers to a state of depressed consciousness, usually obtained through IV analgesia therapy. The technique allows a child to be both pain-free and sedated for a procedure. The child is monitored throughout the process by a nurse. PCA is a pump that delivers pain medication and allows the client to receive medication via continuous infusion or bolus dose. General anesthesia means the client loses all reflexes. This is not necessary for an MRI and it would have to be administered and monitored via an anesthesiologist. An IM injection is painful and frightens the child. It is not necessary when oral and IV medications can be used.

A child needs a peripheral IV start as well as a venous blood sample for a laboratory test. The nurse will take what action?

Coordinate placing the peripheral IV and the lab blood draw. Explanation Coordinate the IV placement and lab blood draw to minimize the number of venipunctures for the child. Gaining venous access for each purpose separately does not do this and is not necessary. Having a well-hydrated child makes venous access easier, but oral hydration will take some time, thus delaying needed treatment.

The nurse is administering medications to the child with congestive heart failure (CHF). Large doses of what medication are used initially in the treatment of CHF to attain a therapeutic level?

Digoxin

The nurse is preparing to give a 4-month-old an oral medication. Which technique demonstrates the nurse's accurate knowledge of the infant's developmental level?

Position the infant upright, offer the infant a bottle of formula, remove the bottle and squirt the medication on the side of the tongue toward the cheek, then offer the infant the bottle again.

The nurse is collecting data on a child with a diagnosis of atopic dermatitis. While interviewing the caregiver, the nurse will direct questions to the caregiver recognizing that which common allergens are involved in eczema? Select all that apply.

Cow's milk Animal dander Nylon

A child is being evaluated for renal and urinary tract disease. What would the nurse expect to be ordered to evaluate the child's glomerular filtration rate? -Urinalysis -Creatinine clearance rate -Kidneys, ureter, and bladder x-ray -Computed tomography scan

Creatinine clearance rate The glomerular filtration rate is measured by creatinine clearance rate, or the amount of creatinine excreted in 24 hours as determined by a 24-hour urine sample along with a venous blood sample and compared with the urine findings. Urinalysis provides general information about kidney function. A kidneys, ureters, and bladder x-ray provides information about the size and contour of the kidneys. A computed tomography reveals the size and density of kidney structures and adequacy of urine flow.

A nurse is assessing the pain level of an infant. Which finding is not a typical physiologic indicator of pain?

Decreased heart rate

A parent is asking how to help the child deal with the peer ridicule at school in regards to enuresis. What is the best response by the nurse? -Demonstrate love and acceptance at home. -Discuss how the child can continue to go to the bathroom instead of in the underwear. -Take away a toy every time the child urinates in his or her pants.- -Demonstrate how to urinate in the bathroom every time the child has an occurence.

Demonstrate love and acceptance at home. Enuresis is the contined incontinence of urine past the age of toliet training. It is a source of shame and embarrassment. It affects the child's life emotionally, behaviorally and socially. It causes the child to have a low self-esteem. Demonstrating love and acceptance at home will help counteract the ridicule the child is getting at school.The child should not be punished for a behavior he or she cannot control. Demonstrating how to use the toliet and going to the bathroom to void are good subjects but they do not help a child who has no control of the enuresis. Testing may need to be done to see if there are anatomical reasons and medications may be needed to correct the problem.

A child is scheduled for a urea breath test. The nurse understands that this test is being performed for which reason? a) Detect Helicobacter pylori b) Evaluate gastric pH c) Confirm pancreatitis d) Determine esophageal contractility

Detect Helicobacter pylori Urea breath test is used to detect the presence of H. pylori in the exhaled breath. This test does not evaluate gastric pH. Serum amylase and lipase levels are used to confirm pancreatitis. Esophageal manometry is used to evaluate esophageal contractile activity and effectiveness. (less)

The nurse is aware of the special needs of children related to pain assessment. What is the priority for the nurse to consider when completing a pain assessment?

Developmental age of child Although all of the options are important for assessing pain in children, the priority to provide an appropriate pain assessment is knowing the developmental age of the child. The chronological and developmental ages may differ and care needs to be based on both, but the type of pain assessment tool used will be based on the developmental age. For children who are nonverbal the nurse needs to also consider the parent's statement of pain in the child.

An adolescent with tinea versicolor is admitted for treatment of the disorder. Which nursing diagnosis will the nurse identify as having the highest priority for this client?

Disturbed body image

A client is experiencing nociceptive pain as a result of a cancerous tumor of the bladder that has metastasized to other organs. What types of pain does the nurse expect the client to report?

Dull Deep aching Sharp stabbing

The nurse is caring for a child with a partial-thickness burn. What assessment findings would the nurse expect to observe?

Edema with wet blistering skin Partial-thickness burns are very painful and edematous and have a wet appearance or the presence of blisters. Full-thickness burns appear red, edematous, leathery, dry, or waxy and may display red or charred skin (eschar).

A nurse is performing postoperative care on a child with a ureteral stent. Which intervention will help manage bladder spasms? -Allow tubes to dangle freely to encourage flow. -Encourage high fluid intake. -Increase low-fat foods. -Apply antibiotic ointment to tube site.

Encourage high fluid intake. Prevent bladder stimulation secondary to a full rectum by completing a preoperative bowel evacuation, encouraging a high fluid intake, promoting early ambulation postoperatively, and administering a stool softener or glycerin suppository postoperatively.

A nurse is developing a teaching plan for the parents of an 8-year-old experiencing nocturnal enuresis. The nurse determines that additional teaching is needed when the parents identify what as an appropriate measure? -Encouraging fluid intake after dinner -Practicing bladder-stretching exercises -Giving desmopressin intranasally -Engaging the child in stress reduction measures

Encouraging fluid intake after dinner In many children, limiting fluids after dinner can be helpful for nocturnal enuresis. Bladder-stretching exercises also can be helpful. If these measures are ineffective, desmopressin may be prescribed. Stress factors may be contributing to the child's problem. Therefore, measures to address stress and promote coping would be appropriate.

The nurse is assisting with the administration of the child's initial dose of parenteral opioids. Which action should the nurse take first?

Ensure naloxone is readily available When administering parenteral or epidural opioids, the nurse should always have naloxone readily available in order to reverse the opioids effects, should respiratory distress occur. Premedication with acetaminophen is not required with opioids. After administration, the nurse should continually assess for adverse reaction. The nurse should assess bowel sounds for decreased peristalsis after administration.

The labor and delivery nurse is caring for a mother who has demonstrated polyhydramnios upon delivery. The newborn displays copious, frothy bubbles of mucus in the mouth and nose, as well as drooling. The nurse is concerned that the infant has what disorder? a) Hiatal hernia b) Gastroschisis c) Esophageal atresia d) Omphalocele

Esophageal atresia Esophageal atresia refers to a congenitally interrupted esophagus where the proximal and distal ends do not communicate; the upper esophageal segment ends in a blind pouch and the lower segment ends a variable distance above the diaphragm. Polyhydramnios is often the first sign of esophageal atresia because the fetus cannot swallow and absorb amniotic fluid in utero, leading to accumulation. Omphalocele and gastroschisis are congenital anomalies of the anterior abdominal wall. Hiatal hernia involves a weakened diaphragm.

A nurse taking a health history of a newborn notes that there is a maternal history of polyhydramnios. What GI condition might this history precipitate? a) Pyloric stenosis b) Cleft palate c) Esophageal atresia (EA) d) Hernia

Esophageal atresia (EA) A maternal history of polyhydramnios is usually present in one-third of cases of EA and in some cases of tracheoesophageal fistula (TEF).

A nurse taking a health history of a newborn notes that there is a maternal history of polyhydramnios. What GI condition might this history precipitate? a) Cleft palate b) Pyloric stenosis c) Hernia d) Esophageal atresia (EA)

Esophageal atresia (EA) Correct Explanation: A maternal history of polyhydramnios is usually present in one-third of cases of EA and in some cases of tracheoesophageal fistula (TEF).

A 3-year-old client is being admitted for a tonsillectomy. The nurse notes the client is fussy, crying, and appears nervous about the procedure. Which action by the nurse will be most helpful in alleviating the child's anxiety?

Explain the procedure to the child using dolls and medical equipment. The nurse will explain the procedure to the client using dolls and medical equipment to help the child understand what will happen. This is most appropriate for a client this age. It is appropriate to provide a tour of the operating room, but not show a video due to the child's developmental age. Deep-breathing exercises are not appropriate for a preschool-age client, nor is having another client talk with the child.

The nurse is preparing to assess the postsurgical pain level of a 6-year-old boy. The child has appeared unwilling or unable to accurately report his pain level. Which assessment tool is most appropriate for this child?

FLACC Behavioral scale The FLACC behavioral scale is a behavioral assessment tool that is useful in assessing a child's pain when the child is unable to report accurately his or her level of pain or discomfort and is reliable for children from age 2 months to 7 years. The preferred base age for the visual analog and numerical scales is 7 years . The FACES pain rating scale and Oucher pain rating scale are appropriate for children as young as 3; however, in this situation the FLACC is required due to the child's inability to report his level of pain.

The nurse is caring for a 12-year-old with cerebral palsy who is unable to communicate verbally. Which pain assessment tool is the most appropriate for the nurse to use when assessing pain in this client?

Face, leg, activity, cry, and consolability (FLACC) descriptors

Inguinal hernia usually occurs in girls. a) True b) False

False Inguinal hernia is a protrusion of a section of the bowel into the inguinal ring. It occurs usually in boys (9:1) because, as the testes descend from the abdominal cavity into the scrotum late in fetal life, a fold of parietal peritoneum also descends, forming a tube from the abdomen to the scrotum.

A nurse admits an infant with a possible diagnosis of congestive heart failure. Which signs or symptoms would the infant most likely be exhibiting?

Feeding problems The indications of CHF vary in children of different ages. Signs in the infant may be hard to detect because they are subtle, but in infants, feeding problems are often seen. In infants and older children, tachycardia is one of the first signs of CHF. In a child with CHF, tachypnea would be seen, not bradypnea. The heart beats faster in an attempt to increase blood flow. Failure to gain weight, weakness, and an enlarged liver and heart are other possible indicators of CHF but are not as common as tachycardia and may take longer to develop.

What method would the nurse use to teach an 8-year-old client how to swallow medications?

Have the child practice swallowing an ice chip. Explanation: Hiding the pill in applesauce or crushing it may help the child swallow it easier, but it does not teach the child how to swallow a pill. The nurse should have the child practice swallowing a small piece of ice, as it will melt and not get stuck in the throat. It is best to put the pill as far back on the tongue to make it easier to swallow.

A 5-year-old child has been admitted to the hospital and is going to have an IV started in the procedure room. Which instructions will be most helpful for the child and the parent?

Have the parent sing softly to the child during the procedure.

The pediatric nurse recognizes that what statement is true regarding medications administered via the intravenous route?

Giving medications through the intravenous route is less traumatic than multiple injections. Explanation: Delivering medications intravenously is actually less traumatic than administering multiple injections. An injection into the fatty tissue between the skin and the muscle is a subcutaneous injection. Medication absorption is quickest via an IV route. When performed properly, all routes of medication administration are safe.

The nurse performs a cardiac assessment and notes a loud heart murmur with a precordial thrill. This murmur would be classified as a:

Grade IV: Explaination: Grade I= is soft and hard to hear. Grade II= is soft and easily heard. Grade III= is loud without thrill Grade IV= A heart murmur characterized as loud with a precordial thrill is classified as Grade IV.

The nurse is conducting a physical examination of an infant with suspected pyloric stenosis. Which finding indicates pyloric stenosis? a) Perianal fissures and skin tags b) Hard, moveable "olive-like mass" in the upper right quadrant c) Sausage-shaped mass in the upper mid abdomen d) Abdominal pain and irritability

Hard, moveable "olive-like mass" in the upper right quadrant A hard, moveable "olive-like mass" in the right upper quadrant is the hypertrophied pylorus. A sausage-shaped mass in the upper mid abdomen is the hallmark of intussusception. Perianal fissures and skin tags are typical with Crohn disease. Abdominal pain and irritability is common with pyloric stenosis but are seen with many other conditions.

----

He needs to take his medicine or he will lose a privilege." The nurse should emphasize that the parents should never threaten the child in order to make him take his medication. It is more appropriate to develop a cooperative approach that will elicit the child's cooperation since he needs ongoing, daily medication. The other statements are correct.

An adolescent boy is diagnosed with hepatitis A. Which problem should be considered when planning care? a) He will be very irritable and perhaps require sedation. b) Hypothermia is common. c) He will become fatigued easily. d) His urine will be dark and infectious.

He will become fatigued easily. Most children with hepatitis are exhausted. Urine is not infectious.

An adolescent boy is diagnosed with hepatitis A. Which problem should be considered when planning care? a) Hypothermia is common. b) He will be very irritable and perhaps require sedation. c) He will become fatigued easily. d) His urine will be dark and infectious.

He will become fatigued easily. Most children with hepatitis are exhausted. Urine is not infectious.

The nurse in a pediatric cardiovascular clinic is talking with the father of a 5-year-old child who underwent cardiac surgery for a heart defect at the age of 3. The father reports that the child has been having increased shortness of breath, tires easily after playing, and has been gaining weight. The nurse is aware that the child is most likely demonstrating symptoms of which acquired cardiovascular disorder?

Heart failure Explanation: Infective endocarditis = would present with intermittent, unexplained low-grade fever, fatigue, anorexia, weight loss, or flu-like symptoms. cardiomyopathy= include respiratory distress, fatigue, poor growth (dilated), chest pain, dizziness, and syncope. Kawasaki Disease= Abdominal pain, joint pain, fever, irritability

What accurately depicts the hemodynamic changes that occur in the body within the first 24 to 48 hours after a burn?

Hematocrit and WBC counts elevate

A 10-year-old male presents with low-grade fever, nausea, and abdominal pain. The nurse examining him suspects appendicitis and checks for rebound tenderness in what quadrant? a) Lower right b) Upper left c) Upper right d) Lower left

Lower right With appendicitis, percussion reveals irritation and pain in the right lower quadrant. Rebound tenderness present with palpation in the right lower quadrant, is referred to as the McBurney point, an area of tenderness 1.5 to 2 in. (3.8 to 5 cm) in from the right anterior superior iliac spine along a line extending to the umbilicus.

A nurse prepares a menu for a client with Crohn disease. What is the focus of dietary management for this disease? a) High calorie, high fiber b) Low calorie, high carbohydrate c) High carbohydrate, high protein d) Low fiber, low calorie

High carbohydrate, high protein The goal of nutritional intervention is to provide adequate nutrient intake to optimize normal growth and development—including pubertal development, which is frequently delayed—and to prevent and correct nutrient deficiencies. Adequate nutrition with a high-protein and high-carbohydrate diet may be recommended

The nurse is caring for a child diagnosed with hydronephrosis. Which manifestation is consistent with complications of the disorder? -Hypertension -Hypotension -Hypothermia -Tachycardia

Hypertension Complications of hydronephrosis include renal insufficiency, hypertension, and eventually renal failure. Hypotension, hypothermia, and tachycardia are not associated with hydronephrosis.

On assessment immediately following cardiac surgery, which condition would the nurse expect to find in an infant?

Hypothermia Cardiac surgery is often performed under hypothermia to decrease the child's oxygen needs during surgery. The postoperative care nurse should assess the infant's vital signs continuously via monitoring. The temperature should be assessed at least once per hour until an optimal temperature is achieved. The infant would have received IV fluids during surgery so hypovolemia should not be the primary concern. The infant will be sleepy from anesthesia, not hyperexcited. Hypertension, if any, has been monitored throughout the surgery and controlled.

A 10-year-old has been bitten on the lower posterior arm by a dog, requiring several stitches. The child was just admitted to the hospital for 3 days of antibiotic therapy. When developing the care plan, the nurse identifies which nursing diagnoses as being the top 2 priorities?

Impaired skin integrity Risk for infection

The nurse is assessing a child who was brought into the clinic. The nurse notes honey-colored crusting on the toddler's face, as seen in the figure. The nurse recognizes this to be what type of infection?

Impetigo Impetigo is a readily recognizable skin rash that is characterized with honey-colored crusting.

The nurse is caring for a child with a diagnosis of pyloric stenosis during the preoperative phase of the child's treatment. What is the highest priority at this time? a) Improving hydration b) Promoting comfort c) Maintaining skin integrity d) Preparing family for home care

Improving hydration Preoperatively the highest priority for the child with pyloric stenosis is to improve nutrition and hydration. Maintaining mouth and skin integrity, and relieving family anxiety are important, but these are not the priority. The child will not likely have intense pain. Preparing the family for home care would be a postoperative goal.

An 8-month-old infant has a ventricular septal defect. Which nursing diagnosis would best apply?

Ineffective tissue perfusion related to inefficiency of the heart as a pump Explanation: A ventricular septal defect permits blood to flow across an opening between the right and left ventricles. It results in increased pulmonary blood flow, but it does not cause cyanosis. The blood in the left ventricle, which flows back into the right ventricle, is already oxygenated. Anytime there is an opening between the heart's ventricles, the heart is not as effective as a pump because the pressure gradients are changed. A ventricular septal defect will not cause respiratory problems or problems with peripheral circulation.

A nurse is applying EMLA as ordered. The nurse understands that EMLA is contraindicated in which situation?

Infants less than 12 months of age receiving methemoglobin-inducing agents EMLA is contraindicated in children less than 12 months who are receiving methemoglobin-inducing agents, such as sulfonamides, phenytoin, phenobarbital, and acetaminophen. Children with darker skin may require longer application times to ensure effectiveness. EMLA is not contraindicated for children less than 6 weeks of age or those undergoing venous cannulation or intramuscular injections.

A 6-year-old girl is diagnosed with pulmonary stenosis. The mother asks the nurse what the likely treatment for this condition will involve. What should the nurse tell the mother?

Insertion of a catheter with an uninflated balloon tip into the affected valve, followed by inflation of the balloon to break up adhesions Explanation: Balloon angioplasty by way of cardiac catheterization is the procedure of choice for pulmonary stenosis. With this procedure, a catheter with an uninflated balloon at its tip is inserted and passed through the heart into the stenosed valve. As the balloon is inflated, it breaks valve adhesions and relieves the stenosis. The other answers refer to interventions related to patent ductus arteriosus, not pulmonary stenosis.

A 9-month-old girl is brought to the emergency room with what appears to be bouts of intense abdominal pain 15 minutes apart in which she draws up her legs and cries, often accompanied by vomiting. In between the bouts, the child recovers and appears to be without symptoms. Blood is found in the stool. What condition should the nurse suspect in this case? a) Volvulus with malrotation b) Intussusception c) Short-bowel/short-gut syndrome d) Necrotizing enterocolitis

Intussusception Intussusception, the invagination of one portion of the intestine into another, usually occurs in the second half of the first year of life. Children with this disorder suddenly draw up their legs and cry as if they are in severe pain; they may vomit. After the peristaltic wave that caused the discomfort passes, they are symptom-free and play happily. In approximately 15 minutes, however, the same phenomenon of intense abdominal pain strikes again. After approximately 12 hours, blood appears in the stool and possibly in vomitus, described as a "currant jelly" appearance. Volvulus with malrotation and necrotizing entercolitis typically occur in the first 6 months of life and do not match the symptoms described above. Short-bowel/short-gut syndrome typically occurs when a large portion of the intestine has been removed due to a previous disease or trauma.

An adolescent girl is going to be treated for a severe case of acne vulgaris. A pregnancy test should be done prior to the adolescent starting treatment with:

Isotretinoin (Accutane) Isotretinoin is a pregnancy category X drug: it must not be used at all during pregnancy because of serious risk of fetal abnormalities.

A nurse reads the medical history of a client who is scheduled for a hernia repair that is termed "reducible." What best describes this type of hernia? a) Its contents can be easily manipulated back into the peritoneal cavity. b) The herniated intestines are twisted and edematous. c) Intestinal obstruction and ischemia may occur. d) The abdominal contents have become trapped.

Its contents can be easily manipulated back into the peritoneal cavity. A hernia in the abdominal region is considered reducible when its contents are easily manipulated back into the peritoneal cavity. An incarcerated hernia occurs when the abdominal contents become trapped and difficult to reduce. A strangulated hernia occurs when the herniated intestines become twisted and edematous compromising blood flow. Intestinal obstruction and ischemia may occur.

A mother brings her 2-week-old newborn to the doctor's office because the child has been experiencing gastroesophageal reflux over the past week. Which interventions should the nurse recommend to the mother at this point? Select all that apply. a) Keep the infant upright in an infant chair for 30 minutes after feeding. b) Feed the infant a formula thickened with rice cereal. c) If breastfeeding, switch to formula. d) Feed the infant while holding her in an upright position. e) Consult a pediatric surgeon regarding having a myotomy procedure performed. f) Consult the physician regarding having botulinum toxin injected into the lower esophageal sphincter.

Keep the infant upright in an infant chair for 30 minutes after feeding. • Feed the infant a formula thickened with rice cereal. • Feed the infant while holding her in an upright position. The traditional treatment of GI reflux is to feed infants a formula thickened with rice cereal (1 tbsp of cereal per 1 oz of formula or breast milk) while holding them in an upright position and then keeping them upright in an infant chair for 30 minutes after feeding so gravity can help prevent reflux. There is no need for the mother to switch from breastfeeding to formula. Injection of botulinum toxin into the lower esophageal sphincter and a myotomy procedure are interventions that would be considered only if the problem does not disappear with feeding solid food and maintaining the child in a more upright position during and following feeding; they would not be appropriate at this point.

The nurse is conducting a physical examination of a boy with erythema multiforme. Which assessment finding should the nurse expect?

Lesions over the hands and feet, and extensor surfaces of the extremities with spread to the trunk Erythema multiforme typically manifests in lesions over the hands and feet, and extensor surfaces of the extremities with spread to the trunk. Thick or flaky/greasy yellow scales are signs of seborrhea. Silvery or yellow-white scale plaques and sharply demarcated borders define psoriasis. Superficial tan or hypopigmented oval-shaped scaly lesions specially on upper back and chest and proximal arms are indicative of tinea versicolor.

The nurse is administering an oral liquid medication to a 5-year-old child. What would be the most appropriate for the nurse to do when administering this medication?

Let the child hold the medication cup. Explanation: Droppers and oral syringes can be used to administer medications to infants and young children. Medication cups and spoons can be used to administer liquid medications to the older child. The child can hold the medication cup and drink the liquid medication. Depending upon the age of the child, he or she may still prefer to take liquid medications via the syringe. It makes taking the medication fun when the child can squirt it into the mouth by himself or herself. The child who is lying down when being given medications should have the head of the bed elevated to at least 45 degrees A 5-year-old child does not need to be restrained for medication administration.

where is Mcburney's Point

McBurney's point is the area in the right lower quadrant of the abdomen where the most pressure tends to be felt upon palpation when the client has appendicitis. It lies between the naval and the right anterior superior iliac spine.

A child with a cardiac structural defect is receiving oxygen therapy. In which position should the child be placed to promote optimal benefits?

Semi-Fowler Explanation: Due to the hemodynamic changes accompanying the underlying structural defect, oxygenation is key. Provide frequent ongoing assessment of the child's cardiopulmonary status. Assess airway patency and suction as needed. Position the child in the Fowler or semi-Fowler position to facilitate lung expansion.

An adolescent is diagnosed with a trichomonal infection. Which medication would the nurse include when teaching the adolescent about treatment for this infection? -Metronidazole -Miconazole -Doxycycline -Acyclovir -Ceftriaxone

Metronidazole Metronidazole is used to treat a trichomonal infection. Miconazole is used to treat candidiasis. Doxycycline is used to treat a chlamydial infection. Acyclovir is used to treat herpes genitalis. Ceftriaxone is used to treat gonorrhea.

The nurse is auscultating heart sounds of a child with a mitral valve prolapse. The nurse should expect which assessment finding?

Mild to late ejection click at the apex Explanation: A mild to late ejection click at the apex is typical of a mitral valve prolapse. Abnormal splitting or intensifying of S2 sounds occurs in children with heart problems, not mitral valve prolapse. Clicks on the upper left sternal border are related to the pulmonary area.

The parents of a 6-month-old have brought their child to the emergency department with vomiting and diarrhea for the past 3 days. The report the child as being very lethargic today. During the assessment the nurse notes decreased skin turgor, delayed capillary refill, and pale, slightly dry skin. Based on the objective and subjective data, what does the nurse determine the child to be? a) Moderately dehydrated b) Severely dehydrated c) Mildly dehydrated d) Well hydrated

Moderately dehydrated In addition to these signs and symptoms, signs and symptoms of moderate dehydration also include sunken fontanels, mildly sunken orbits, and urine output

The parents of a 6-month-old have brought their child to the emergency department with vomiting and diarrhea for the past 3 days. The report the child as being very lethargic today. During the assessment the nurse notes decreased skin turgor, delayed capillary refill, and pale, slightly dry skin. Based on the objective and subjective data, what does the nurse determine the child to be? a) Mildly dehydrated b) Moderately dehydrated c) Severely dehydrated d) Well hydrated

Moderately dehydrated In addition to these signs and symptoms, signs and symptoms of moderate dehydration also include sunken fontanels, mildly sunken orbits, and urine output

A nurse is performing postoperative care on a child with a ureteral stent. Which intervention will help manage tube patency? -Monitor output. -Allow tubes to dangle freely to encourage flow. -Maintain fluid restriction. -Provide a low-sodium diet.

Monitor output. A ureteral stint is placed in the ureter temporarily to aid in the drainage of urine. It is removed via cystoscopy when it is time for discontinuation. The nurse should monitor output cafefully when a ureteral stint is in place. This is an indication that the stent is patent and functioning properly. The tubes are inserted into the ureter so they would not dangle on the outside of the body. There is no need to maintain fluid restriction or a low-sodium diet just because of the stent. This would only be necessary if there were other disease processes affecting the child.

The nurse is caring for a pediatric client following an open appendectomy. The client rates the pain an "8" on a 0 to 10 pain scale and the nurse administers morphine sulfate intravenously to the client per the primary health care provider's prescription. Which nursing action is priority following administration of the medication?

Monitor the client's respiratory status.

The nurse is caring for a pediatric client following an open appendectomy. The client rates the pain an "8" on a 0 to 10 pain scale and the nurse administers morphine sulfate intravenously to the client per the primary health care provider's prescription. Which nursing action is priority following administration of the medication?

Monitor the client's respiratory status. It is priority for the nurse to assess the client's respiratory status after administering a narcotic medication. The nurse would reassess the client's pain level and document; however, these are not priority over monitoring the respiratory status. Playing a game may help distract the

The nurse is reviewing the medical record of a child with a cleft lip and palate. When reviewing the child's history, what would the nurse identify as a risk factor for this condition? a) Maternal use of acetaminophen in third trimester b) Mother age 42 with pregnancy c) Preterm birth d) History of hypoxia at birth

Mother age 42 with pregnancy Advanced maternal age is a risk factor for cleft lip and palate. Drugs such as anticonvulsants, steroids, and other medications during early pregnancy are considered risk factors. Acetaminophen is not associated with an increased risk for cleft lip and palate. Preterm birth is not a risk factor for the development of cleft lip and palate. Hypoxia or anoxia is a risk factor for the development of necrotizing enterocolitis.

Parents asks the nurse why their premature infant is receiving a feeding through the mouth rather than the nose. What is the best explanation by the nurse?

Newborns are obligate nose breathers so nasogastric may obstruct their breathing. Explanation: Whether enteral catheters should be passed through the nares or the mouth is controversial. Because newborns are obligate nasal breathers, passing a catheter through the nose may obstruct their breathing space, and repeated insertion of a nasogastric tube can cause inflammation and obstruction of the nose; thus most tubes are inserted orally in small infants. Orogastric insertion can also decrease the possibility of striking the vagal nerve in the back of the throat and causing bradycardia, whereas nasogastric tubes increase the possibility of striking the vagal nerve.

A 3-month-old girl is found to have an umbilical hernia at a well visit. On examination, the nurse discovers that the fascial ring through which the intestine protrudes is about 1 cm in diameter. Which of the following should she mention to the girl's father as the likely intervention required to correct this condition? a) No intervention is needed, as the opening will most likely close spontaneously b) Taping a silver dollar over the area will help reduce the hernia c) Surgery at age 1 to 2 years will likely be needed to repair the condition d) Wrapping an elastic band around the child's waist should correct the problem

No intervention is needed, as the opening will most likely close spontaneously An umbilical hernia is a protrusion of a portion of the intestine through the umbilical ring, muscle, and fascia surrounding the umbilical cord. If the fascial ring through which the intestine protrudes is less than 2 cm, closure will usually occur spontaneously after the child begins to walk so no repair of the disorder will be necessary. If the fascial ring is larger than 2 cm, ambulatory surgery for repair is generally indicated to prevent herniation and intestinal obstruction or bowel strangulation. This is usually done at 1 to 2 years of age. Some parents believe holding an umbilical hernia in place by using "belly bands" or taping a silver dollar over the area will help reduce the hernia. These actions can actually lead to bowel strangulation so should be avoided.

The nurse is caring for a 5-year-old child who underwent a painful surgical procedure earlier in the day. The nurse notes the child has not reported pain to any of the nursing staff. Which action by the nurse is indicated?

Observe for behavioral cues consistent with pain

While working in the emergency room, the nurse receives a call that a 3-year-old child sustained extensive burns in a house fire. Assuming all of the following actions are included in the standing burn-care protocol, which action should be the nurse perform first?

Obtain a weight. Explanation: A burn victim will require large amounts of fluid hydration to replace fluid losses. Obtaining a weight provides a base for calculating the fluid that will need to be replaced. Nasogastric tube placement and/or drinking milk are not actions to take at this point. Tetanus can be given later and is not critical to active management.

A mother calls the doctor's office and tells the nurse that she is concerned because her 4-month-old keeps "spitting up" with every feeding. What would indicate that the child is regurgitating as opposed to vomiting? a) Is projected 1 ft away from infant b) Only occurs with feeding c) Continues until stomach is empty d) Is curdled and extremely sour smelling

Only occurs with feeding Regurgitation typically occurs only with feeding, runs out of the mouth with little force, smells barely sour and is only slightly curdled, appears to cause no pain or distress, occurs only once per feeding, and amounts to only about 1 to 2 tsp. Vomiting may occur at times other than feeding, is forceful and is typically projected 1 ft or more away from infant, is extremely sour smelling and curdled, is typically accompanied by crying, continues until the stomach is empty, and amounts to the full stomach contents.

The nurse is caring for a teenager diagnosed with acute pancreatitis. Which order would the nurse question? a) Nasogastric tube placed to suction b) Serum amylase levels c) NPO d) PO pain management

PO pain management Maintain NPO status and nasogastric tube suction and patency. Administer intravenous fluids to keep the child hydrated and correct any alterations in fluid and electrolyte balance. Pain management is crucial in children with pancreatitis, due to NPO status, medications are typically prescribed intravenously. Serial monitoring of serum amylase levels will determine when oral feeding may be restarted.

A newborn has been diagnosed with a congenital heart disease. Which congenital heart disease is associated with cyanosis?

Tetralogy of Fallot Explanation: Tetralogy of Fallot is associated with cyanosis. The defects include ventricular septal defect (VSD), right ventricular hypertrophy, right outflow obstruction, and overriding aorta. Coarctation of aorta, pulmonary stenosis, and aortic stenosis are acyanotic heart diseases and are not associated with cyanosis.

A mother brings her 10-year-old son to the ER with complaints of abdominal pain. The nurse performing a physical assessment notes the following symptoms: upper right quadrant pain that radiates to the back; fever; nausea; and abdominal distention. Which disease would the nurse consider as a diagnosis? a) Crohn disease b) Ulcerative colitis c) Appendicitis d) Pancreatitis

Pancreatitis The child admitted with the suspicion of pancreatitis typically has a complaint of abdominal pain, either epigastric, upper left, or upper right quadrant pain that may radiate to the back. Nausea and vomiting, fever, tachycardia, hypotension, and jaundice may be present. Abdominal signs such as abdominal distention, decreased bowel sounds, rebound tenderness, and guarding also may be noted.

The nurse is obtaining the history of an infant with a suspected intestinal obstruction. Which response regarding newborn stool patterns would indicate a need for further evaluation for Hirschsprung disease? a) Has had diarrhea for 3 days b) Passed a meconium stool in the first 24 to 48 hours of life c) Passed a meconium plug d) Constipated and passing gas for 2 days

Passed a meconium plug If the parent reports that the child passed a meconium plug, the infant should be evaluated for Hirschsprung disease. Constipation, not diarrhea, is associated with this condition; however, constipation alone would not necessarily warrant further evaluation for Hirschsprung disease. Passing a meconium stool in the first 24 to 48 hours of life is normal.

The nurse is discussing dietary intake with the parents of a 4-year-old child who has been diagnosed with atopic dermatitis. Later, the nurse notes the menu selection made by the parents for the child. Which selection indicates the need for further instruction?

Peanut butter and jelly sandwich Atopic dermatitis is commonly associated allergies to food. Common culprits may include peanuts, eggs, orange juice, and wheat-containing products.

A nurse is caring for a child with Kawasaki disease. Which assessment finding would the nurse expect to see?

Peeling hands and feet; fever Explanation: Kawasaki disease is an acute systemic vasculitis. Symptoms begin with very high fevers. One of the signs of Kawasaki disease is the peeling hands and feet and in perineal region. The child is usually tachycardic and laboratory values would indicate increased platelets and decreased hemoglobin. Another classic sign of Kawasaki is the strawberry tongue. The other symptoms are not necessarily characteristic of Kawasaki disease. The child should be evaluated if there are impalpable pulses because this could indicate a heart defect or some other serious illness.

The nurse is caring for a 6-year-old with a congenital heart defect. To best relieve a hypercyanotic spell, what action would be the priority?

Place the child in a knee-to-chest position. Explanation: The priority nursing action is to place the infant or child in a knee-to-chest position. Once the child has been placed in this position, the nurse should provide supplemental oxygen or administer medication as ordered. A calm, comforting approach should be used but is not the priority action. Once a child is placed in the knee-to-chest position, supplemental oxygen would be provided as ordered.

An infant with tetralogy of Fallot becomes cyanotic. Which nursing intervention would be the first priority?

Place the infant in the knee-chest position. Explanation: Placing the infant in the knee-chest position is the first priority when caring for an infant with tetralogy of Fallot. Starting IV fluids and preparing the child for surgery would not be necessary since it is known that the infant has a cyanotic birth defect. Raising the head of the bed would not be a priority since the infant needs to be placed in the knee-chest position.

A child is having difficulty swallowing pills. What is the best action for the nurse to take to help this child swallow medications?

Place the pills in a bite of ice cream or applesauce. Explanation: The most useful technique when children cannot swallow pills is to put them into some ice cream or applesauce. This allows the medication to be administered in the original form. The nurse should not use candy for practice, because this may suggest to the child that medicine is the same as candy. The nurse should never crush medications which are enteric coated or time released. The nurse shoul

When reviewing the record of a child with tetralogy of Fallot, what would the nurse expect to discover?

Polycythemia Explanation: Tetralogy of Fallot= is a congenital heart defect causing decreased pulmonary blood flow. This causes mild to severe oxygen desaturation. To compensate for the low blood oxygenation the kidneys produce erythropoietin to stimulate the bone marrow to make more red blood cells (RBCs). The increased amount of RBCs is known as polycythemia. This increased blood volume causes more workload on the heart. It also does not increase the amount of blood reaching the lungs so the child remains desaturated. Leukopenia, increased platelets, and anemia are not associated with tetralogy of Fallot.

The emergency department nurse is assessing a child who has presented with a 2-day history of nausea and vomiting with pain that is isolated to the right upper quadrant of the abdomen. Which action is most appropriate? a) Assess the child's usual urinary voiding pattern b) Administer antacids as ordered c) Prepare the child for admission to the hospital d) Encourage fluid intake

Prepare the child for admission to the hospital The child's presentation is consistent with cholecystitis, which necessitates surgery in most cases. The child should be kept NPO and antacids are of no benefit. Genitourinary involvement is atypical.

The nurse is caring for a 10-year-old boy with end-stage renal disease (ESRD) with metabolic acidosis. What would the nurse expect to administer if ordered? -Sodium bicarbonate tablets -Ferrous sulfate -Vitamin D -Erythropoietin

Sodium bicarbonate tablets Bicitra or sodium bicarbonate tablets are used for the correction of acidosis. Ferrous sulfate is used for the treatment of anemia. Vitamin D and calcium are used for the correction of hypocalcemia and hyperphosphatemia. Erythropoietin stimulates red blood cell growth.

A 12-year-old boy has just undergone a liver transplantation and is recovering. After performing a finger stick puncture and assessing the results, the nurse administers a 10% solution of dextrose IV. What is the correct rationale for this intervention? a) Prevention of T-cell rejection of the transplanted liver b) Prevention of hypoglycemia c) Reduction of hypertension d) Maintenance of electrolyte balance

Prevention of hypoglycemia Hypoglycemia is a major danger following liver transplantation because glucose levels are regulated by the liver, and the transplanted organ may not function efficiently at first. Assess serum glucose levels hourly by finger stick puncture. A 10% solution of dextrose IV may be necessary to prevent hypoglycemia. Careful tissue matching before the transplantation is needed to reduce the possibility of stimulating T-cell rejection. Sodium, potassium, chloride, and calcium levels are evaluated approximately every 6 to 8 hours to be certain electrolyte balance is maintained, but potassium is rarely added to IV solutions because of the risk that renal failure has occurred. IV therapy with hypotensive agents such as hydralazine (Apresoline) and nitroprusside may be needed to reduce hypertension.

The nurse is developing the plan of care for a 3-year-old child diagnosed with atopic dermatitis. Which client outcomes are common focuses for a child with this diagnosis? Select all that apply.

Promotion of skin hydration Maintenance of skin integrity Prevention of infection

The nurse is teaching an in-service program to a group of nurses on the topic of children diagnosed with rheumatic fever. The nurses in the group make the following statements. Which statement is most accurate regarding the diagnosis of rheumatic fever?

Strep throat

A nurse is administering ear drops to a 7-year-old girl. What should the nurse do?

Pull the pinna of the ear up and back to straighten the external ear canal. Explanation: Remind the child ear drops can feel odd, as if someone were tickling the ear. Ear drops must always be used at room temperature or warmed slightly as cold fluid, such as medication taken from a refrigerator, does cause pain and may also cause severe vertigo as it touches the tympanic membrane. If the child is older than 2 years, pull the pinna of the ear up and back. Instill the specified number of drops into the ear canal. Hold the child's head in the sideways position while you count to 60 to ensure the medication fills the entire ear canal.

A parent brings an infant in for poor feeding and listlessness. Which assessment data would most likely indicate a coarctation of the aorta?

Pulses weaker in lower extremities compared to upper extremities Explanation: With coarctation of the aorta there is a narrowing causing the blood flow to be impeded. This produces increased pressure in the areas proximal to the narrowing and a decrease in pressures distal to the narrowing. Thus, the infant would have decreased systemic circulation. The upper half of the body would have an increased B/P and be well perfused with strong pulses. The lower half of the body would have decreased B/P with poorer perfusion and weaker pulses. Coarctation is not a cyanotic defect. The cyanosis would be associated with tetralogy of Fallot.

A mother is alarmed because her 6-week-old boy has begun vomiting almost immediately after every feeding. In the past week, the vomiting has grown more forceful, with the vomit projecting several feet from his mouth. He is always hungry again just after vomiting. At the physician's office, the nurse holds the child and gives him a bottle of water. While he drinks, she notes an olive-size lump in his right abdomen. Which condition should the nurse suspect in this child? a) Appendicitis b) Pyloric stenosis c) Peptic ulcer disease d) Gastroesophageal reflux

Pyloric stenosis With pyloric stenosis, at 4 to 6 weeks of age, infants typically begin to vomit almost immediately after each feeding. The vomiting grows increasingly forceful until it is projectile, possibly projecting as much as 3 to 4 feet. Infants are usually hungry immediately after vomiting because they are not nauseated. A definitive diagnosis can be made by watching the infant drink. If a pyloric stenosis is present, the sphincter feels round and firm, approximately the size of an olive in the right abdomen. Peptic ulcer disease in neonates usually presents with hematemesis (blood in vomitus) or melena (blood in the stool). Gastroesophageal reflux involves a small (1-2 tsp) volume and is not forceful. Appendicitis typically begins with anorexia for 12 to 24 hours; children do not eat and do not act like their usual selves. Nausea and vomiting may then occur, followed by diffuse abdominal pain.

The nurse is examining a child for indications of frostbite and notes blistering with erythema and edema. The nurse notes which degree of frostbite?

Second degree frostbite Second degree frostbite demonstrates blistering with erythema and edema. First degree frostbite results in superficial white plaques with surrounding erythema. In third degree frostbite, the nurse would note hemorrhagic blisters that would progress to tissue necrosis and sloughing when the fourth degree is reached.

In caring for the child with asthma, the nurse recognizes that bronchodilator medications are administered to children with asthma for which reason?

Relief of acute symptoms Explanation: Bronchodilators are used for quick relief of acute exacerbations of asthma symptoms. Mast cell stabilizers help to stabilize the cell membrane by preventing mast cells from releasing the chemical mediators that cause bronchospasm and mucous membrane inflammation. Leukotriene inhibitors are given by mouth along with other asthma medications for long-term control and prevention of mild, persistent asthma. Bronchodilators are not effective for pain.

The mother of 6-month-old girl is concerned about her daughter getting a urinary tract infection. What should the nurse mention to the mother to help prevent this condition? -Report any abnormally colored urine to the child's primary care provider. -Wipe from back to front when changing the girl's diaper. -Discontinue prescribed antibiotics once symptoms of UTI have disappeared. -Bathe the child with bubble bath once a week.

Report any abnormally colored urine to the child's primary care provider. Several important interventions can help prevent urinary and renal disease in children. The first intervention is to educate parents and caregivers about wiping from front to back (not back to front) when changing diapers of female infants. Remind parents of simple ways to prevent UTI, such as not allowing children to bathe with bubble bath. Teach parents to recognize that abnormally colored urine (red, black, or cloudy) should not be dismissed as this could be the beginning of a UTI or kidney disease. Educating parents about the importance of giving the full course of antibiotics prescribed for UTIs can help prevent return reinfection; giving the full course of antibiotics after a streptococcal infection can help prevent acute glomerulonephritis.

A child with gastroenteritis has been unable to keep oral medication down. What nursing intervention would be appropriate for this client?

Request an intravenous form of the medication. Absorption is the transfer of the drug from its point of entry into the bloodstream, and vomiting and diarrhea interfere with absorption because the drug does not remain in the gastrointestinal tract long enough to be absorbed. Distribution is not affected by vomiting and diarrhea, as it involves movement of the drug through the bloodstream. Metabolism involves conversion of the drug into an active or inactive form, and is unaffected by gastroenteritis. Excretion is the elimination of the drug from the body, usually through the kidneys. This is also unaffected by vomiting and diarrhea.

When providing care to a child with vesicoureteral reflux (VUR), which nursing diagnosis would be the priority? -Risk for infection -Excess fluid volume -Imbalanced nutrition less than body requirements -Activity intolerance

Risk for infection When vesicoureteral reflux is present, the primary goal is to avoid urine infection so that infected urine cannot gain access to the kidneys. Fluid volume typically is not a problem associated with VUR. Nutritional problems are not associated with VUR. Activity intolerance is not associated with VUR.

A child with heart disease is receiving digoxin and a diuretic. Which laboratory test result would be most important for the nurse to monitor?

Serum potassium level Explanation: Children receiving diuretics should have serum potassium levels obtained because diuretics tend to deplete the body of potassium. This is even more important if the child is also receiving digoxin because low serum potassium levels potentiate or increase the effect of the drug. Serum sodium levels may be obtained in children with heart failure to ensure that an increased sodium level is not causing edema. The erythrocyte sedimentation rate identifies inflammation and is unrelated to the use of diuretics and digoxin. Oxygen saturation levels may be done to evaluate for hypoxemia, but these results would be unrelated to the use of digoxin and diuretics.

What is the best technique to perform an assessment of the skin?

Skin assessment involves inspection and palpation in a room with natural daylight.

The client is scheduled to have potassium hydroxide testing performed. What will be needed to complete this test?

Skin scrapings Potassium hydroxide (KOH) testing is done to assess for the presence of a fungal infection. Skin scrapings are placed on a microscope slide and a drop of KOH 20% drop is added.

The nurse is taking a health history of a 12-year-old boy presenting with scrotal pain. Which assessment finding would indicate testicular torsion? -Sudden onset of severe scrotal pain with significant hemorrhagic swelling -Enlarged inguinal glands and fever -Hardened and tender epididymitis with edema and erythema of scrotum -Fever, scrotal swelling, and urethral discharge

Sudden onset of severe scrotal pain with significant hemorrhagic swelling Testicular torsion is characterized by a testicle that is abnormally attached to the scrotum and twisted. Signs and symptoms include sudden onset of severe scrotal pain with significant hemorrhagic swelling. Enlarged glands and fever point to infection. A hardened and tender epididymitis points to epididymitis. Fever and urethral discharge suggest infection. Scrotal swelling is associated with testicular torsion, epididymitis, and hydrocele.

Constipation may be initially caused by psychological problems T or F

T

When caring for a child with a congenital heart defect, which assessment finding may be a sign the child is experiencing heart failure?

Tachycardia Why? Lets See... eart failure occurs when the heart has the inability to pump effectively to provide adequate blood, oxygen, and nutrients to the body's organs and tissues. Symptoms occur because of three factors. The neurohormonal influences cause symptoms of tachycardia, pallor, decreased urine output, sweating, hypertension, weight gain and edema. The symptoms seen from systolic dysfunction are dyspnea on exertion, increased work of breathing, and feeding difficulties. Diastolic influences produce hepatomegaly, jugular vein distention and periorbital edema.

A 16-year-old adolescent tells the nurse about having severe dysmenorrhea. Which action would be the best health teaching measure? -Take over-the-counter ibuprofen for its prostaglandin action. -Take acetaminophen beginning with the first day of a menstrual flow. -Drink a minimum of fluid if having pain. -Use ice to help in reducing inflammation and pain.

Take over-the-counter ibuprofen for its prostaglandin action. Dysmenorrhea is pain associated with menstruation. A prostaglandin release is responsible for the smooth muscle contraction of the uterus during menstruation. The nonsteroidal anti-inflammatory drug Ibuprofen has an antiprostaglandin mechanisim that will block the prostaglandin release. It is the best choice for dysmenorrhea. Acetominophen has no antiprostaglandin properties, so it is not the drug of choice. Ice will only work on localized areas so it has limitied, if any, effect on the uterus. Ice also is a vasoconstrictor and reduced blood flow could intensify the pain. Fluid intake has no effect on uterine pain.

The caregiver of a child being treated at home for acute glomerulonephritis calls the nurse reporting that her daughter has just had a convulsion. The child is resting comfortably but the caregiver would like to know what to do. The nurse would instruct the caregiver to take which action? -Weigh the child in the same clothes she had been weighed in the day before and report the two weights to the nurse while the nurse is on the phone. -Take the child's blood pressure and report the findings to the nurse while the nurse is still on the phone. -Give the child a diuretic and report back to the nurse in a few hours. -Give the child fluids and report back to the nurse in a few hours.

Take the child's blood pressure and report the findings to the nurse while the nurse is still on the phone. Blood pressure should be monitored regularly using the same arm and a properly fitting cuff. If hypertension develops, a diuretic may help reduce the blood pressure to normal levels. An antihypertensive drug may be added if the diastolic pressure is 90 mm Hg or higher. The concern is immediate so reporting the findings in a few hours could delay needed treatment. The child should be weighed daily in the same clothes and using the same scale, but the blood pressure is the priority in this situation.

The nurse is caring for a child admitted with acute glomerulonephritis. Which clinical manifestation would likely have been noted in the child with this diagnosis? -Loose, dark stools -Tea-colored urine -Strawberry-red tongue -Jaundiced skin

Tea-colored urine The presenting symptom in acute glomerulonephritis is grossly bloody urine. The caregiver may describe the urine as tea or cola colored. Periorbital edema may accompany or precede hematuria. Loose stools are seen in diarrhea. A strawberry-colored tongue is a symptom seen in the child with Kawasaki disease. Jaundiced skin is noted in hepatitis.

To prevent further urinary tract infections in a preschooler, what measures would you teach her mother? -Encourage her to be more ambulatory to increase urine output. -Teach her to take frequent tub baths to clean her perineal area. -Suggest she drink less fluid daily to concentrate urine. -Teach her to wipe her perineum front to back after voiding.

Teach her to wipe her perineum front to back after voiding. Escherichia coli can be easily spread from the rectum to the urinary meatus and cause infection if girls do not take precautions against this.

The nurse admits a child who has sustained a severe burn. The child's immunizations are up to date. Which immunization would the child most likely be given at this time?

Tetanus toxoid vaccine If inoculations are up to date, a booster dose of tetanus toxoid is required to protect the child from infection introduced into the burn.

The nurse is assessing a 10-day-old infant for dehydration. Which finding indicates severe dehydration? a) Pale and slightly dry mucosa b) Soft and flat fontanels c) Tenting of skin d) Blood pressure of 80/42 mm Hg

Tenting of skin Tenting of skin is an indicator of severe dehydration. Soft and flat fontanels indicate mild dehydration. Pale and slightly dry mucosa indicates mild or moderate dehydration. Blood pressure of 80/42 mm Hg is a normal finding for an infant.

The nurse is caring for a 5-month-old boy with an undescended left testis. What would the nurse identify as indicative of true cryptorchidism? -Testis cannot be "milked" down inguinal canal -Fluid detected in scrotal sac -Venous varicosity detected along the spermatic cord -Testis can briefly be brought into scrotum

Testis cannot be "milked" down inguinal canal With true cryptorchidism, the retractile testis cannot be "milked" down the inguinal canal. Fluid in the scrotal sac is a hydrocele. A venous varicosity along the spermatic cord is a varicocele. Testis that can be brought into the scrotum refers to a retractile testis.

The nurse is caring for a 3-year-old child with the surgical repair of hypospadias. The preschooler returned from the postanesthesia care unit with an indwelling urinary catheter. What parental teaching is most helpful? -The catheter insertion site will leave only a minimal scar. -Back pressure from such drainage may result in nephrotic syndrome. -The child must be reevaluated at puberty for testicular function. -The childwill always have tenderness on penile erection.

The catheter insertion site will leave only a minimal scar. Hypospadius is a urethral defect in which the opening is on the ventral surface rather than at the end of the penis. If left untreated it may mean the boy will not be able to void standing as the aim will be different; in addition, it will cause intereference with the deposition of sperm during intercourse. The completed surgery requires the use of a catheter. The catheter, along with the penis, is taped to the abdomen to reduce pressure on the urethral sutures.The tube insertion site will leave only a minimal scar, if any. A hypospadias repair should have no long-term consequences.

The nurse knows this is a description of peritoneal dialysis when compared to hemodialysis: -The child can live a more normal lifestyle. -There are strict diet and fluid restrictions. -Therapy is only 3 to 4 days per week. -The child must go into a facility to get peritoneal dialysis.

The child can live a more normal lifestyle. The child can live a more normal lifestyle with peritoneal dialysis. This is a 7-day-a-week procedure, but there are less diet restrictions and more freedom with this type of procedure. Peritoneal dialysis can be performed at home.

The nurse is collecting data for a child diagnosed with acute glomerulonephritis. What would the nurse likely find in this child's history? -The child has a sibling with the same diagnosis. -The child had a congenital heart defect. -The child recently had an ear infection. -The child is being treated for asthma.

The child recently had an ear infection. In the child with acute glomerulonephritis, presenting symptoms appear 1 to 3 weeks after the onset of a streptococcal infection, such as strep throat, otitis media, tonsillitis, or impetigo. There is not a family history of the disorder, a history of congenital concerns or defects, nor asthma in children with acute glomerulonephritis.

The nurse is caring for a 2-year-old child who has been hospitalized after being injured in an automobile accident. During the assessment the child is quiet and watchful of all the nurse's actions. When considering the level of pain being experienced by the child what inference can be made?

The child's nonverbal behaviors may indicate the presence of discomfort. Responses to pain can vary in children. A child of this age may present with vocal behaviors indicating pain. The child may be tearful or crying loudly. Being quiet can also signal pain.

Which goal of therapy would be appropriate for a nurse to establish with a client's family and a client who has a diagnosis of enuresis? -The child wakes up once during the night for a glass of water. -The client wets only when involved in an activity. -The client remains continent throughout the night. -The parent takes the client to the bathroom at night.

The client remains continent throughout the night. The goal of therapy is for the client to be continent of urine throughout the night. The nurse should encourage the child to awaken and void and not have any fluids before bedtime. During an activity, the child should be encouraged to void before and after the activity to prevent incontinence.

A newborn is diagnosed with hypospadias and the parents want the newborn to be circumcised. What would be the best response by the nurse? -The foreskin is needed for repair. -Circumcision is usually performed after 1 year of age. -Circumcision with a hypospadias will cause meatal stenosis. -The circumcision may predispose the newborn to renal failure.

The foreskin is needed for repair. Hypospadias occurs when the meatal opening is on the ventral surface of the penis rather than at the end of the penis. The newborn with this condition is not circucised at birth because the excess skin may be needed to reconstruct the meatus during surgical repair. Once the hypospadias is repaired, a circumcision can be performed as part of the procedure. Hypospadias repair is usually done after the newborn is 1 year or older. Meatal stenosis has to do with the urethral opening diameter, not the placement. Circumcision or hypospadias repair does not affect the functioning of the renal system so neither would predispose the newborn to renal failure.

The nurse is caring for a child that was dehydrated following gastric surgery but has since been re-hydrated. The physician orders intravenous maintenance fluid rate for the child. How will the nurse determine the intravenous maintenance fluid rate per hour for this child who weighs 40 kg?

The formula to determine maintenance fluid rate is: *100 mL/kg for first 10 kg *50 mL/kg for next 10 kg *20 mL/kg for remaining kg *Add together for total mL needed per 24-hour period. *Divide by 24 for mL/hour fluid requirement. Therefore, for a child weighing 40kg the equation is: *100 X 10= 1000 *50 X 10= 500 *20 X 20= 400 *1000 + 500 + 400= 1900 *1900/24= 79.17= 79 mL/hr

An infant has a surgical repair of a congenital heart defect. In the immediate postoperative period, which scenarios best indicate that the infant is in pain? Select all that apply.

The infant appears restless and wrinkles the face. The best objective data that the infant is experiencing pain include a flexed body position, crying, a wrinkled face, a clenched fist or the inability to find a restful position. Crying when the parent picks up the infant potentially indicates pain or discomfort due to a position change. Physiological changes can also include changes in the infant's vital signs. Infant fatigue may cause a poor suck. Poor interaction may indicate fatigue or a potential bonding issue.

When examining a child with congenital heart disease, an organ in the upper right quadrant of the abdomen can be palpated at 4 cm below the rib cage. What would most likely explain this assessment finding?

The liver size increases in right-sided heart failure. Explanation: The liver increases in size due to right-sided heart failure. This is one of the cardinal signs of congestive heart failure. The spleen is in the upper left quadrant of the abdomen and would increase in size under certain circumstances, but this is asking for the upper right quadrant information. There are certain medications that can affect the liver, but this would not be the most likely reason for hepatomegaly.

The home care nurse is conducting an in-home visit for a child who had corrective surgery for hypospadias 3 days prior. What would alert the home care nurse to provide additional teaching? -The mother indicates the child is fussy, but calms down when she holds him on her hip. -The mother states, "I can't wait until I can bath him the tub again...he enjoys it so much." -The mother expresses relief that the child was not also diagnosed with cryptorchidism at birth. -The mother states, "I have had to buy more diapers since having to double diaper him."

The mother indicates the child is fussy, but calms down when she holds him on her hip. Hypospadias is a condition in which the urethral opening in on the ventral surface of the penis. Surgical repair involves a catheter or stent left in place for 3-7 days postoperatively. Activities or play that involves straddling (such a being carried on mom's hip) are discouraged to prevent trauma to the surgical site and catheter/stent. The child should be double diapered to prevent stool from contaminating the catheter/stent and operative site and causing an infection. The child should not be bathed in a tub until the catheter/stent is removed. Crypotoorchidism is a common diagnosis along with hypospadias.

A nurse is interviewing a mother who is about to give birth. Which response would alert the nurse for a higher potential for a heart defect in the newborn?

The mother states she has lupus. Explanation: A health history should be obtained from the pregnant woman. This history should include having problems during birth of any previous children, frequent infections, chromosomal abnormalities, having a premature birth, having an autoimmune disease or taking long-term medications such as corticosteroids. Lupus= while pregnant could contribute to a congenital heart defect. Acetaminophen & sleeping= do not affect the newborn's potential for developing a heart defect. The seizure medication= can have an impact on the newborn having a heart defect, but not necessarily a history of seizures in the mother. A seizure in the mother would be more related to hypoxia in the newborn than a heart defect.

The nurse is caring for a term neonate suffering from meconium aspiration in the nursery. The nurse reviews orders for a peripherally inserted central catheter (PICC) line placement and intubation. Which statement demonstrates the nurse's knowledge of painful procedures as related to a neonate?

The newborn's pain pathway components are developed enough at birth to experience pain.

The nurse is caring for an infant who has impetigo and is hospitalized. Which nursing intervention is the highest priority for this child?

The nurse follows contact precautions. Impetigo is highly contagious and can spread quickly. The nurse should follow contact (skin and wound) precautions, including wearing a cover gown and gloves. The nurse will soak the crusts with warm water, apply topical antibiotics, and apply elbow restraints, but these are not as high a priority as trying to prevent the spread of the infection by following contact precautions.

The nurse is mentoring a newly graduated nurse on the pediatric unit. Which action by the new nurse requires further instruction when preparing an intramuscular injection for a 6-month-old? Select all that apply.

The nurse prepares to administer 0.7 mL of solution using one injection The nurse prepares to inject the medication into the ventrogluteal site Explanation: No more than 0.5 mL of medication should be administered intramuscularly to an infant, and the preferred site is the vastus lateralis muscle due to muscle development. A 5/8 to 1 inch, 22-25 gauge needle is the preferred range for an infant. Viscosity (thickness) of the medication must be considered when choosing the needle size in order to ensure proper administration.

When administering medications to an infant, what information would be most important for the nurse to consider?

The oral medication should be directed toward the posterior side of the mouth when using a syringe or dropper.] A syringe or dropper should be directed toward the posterior side of the mouth with the infant in the upright position when administering an oral medication.

In understanding the gastrointestinal system, the nurse recognizes that this system includes the stomach and intestines as well as: a) The brain and spinal cord b) The pharynx and esopagus c) Nerves throughout the abdomen d) A protective cushion lining the organs

The pharynx and esopagus The main organs of the gastrointestinal (GI) or digestive system are the mouth, pharynx (throat), esophagus, stomach, small intestine, large intestine, rectum, and anal canal. The brain, spinal column, and nerves are part of the nervous system, and there is a protective coating surrounding the nerves.

The student nurse is preparing to care for a recently placed gastrostomy tube. Which action would prompt further instruction from the overseeing nurse?

The student obtains an antimicrobial soap to clean the area surrounding the tube. Explanation: The skin around a gastrostomy tube requires cleaning at least once a day. Routine site care includes gentle cleansing with sterile water or saline for newly placed tubes, or for established tubes, soap and water followed by rinsing or cleaning with water alone. To clean under an external disc or bumper, a cotton-tipped applicator may be used.

The parent of a child with a central venous catheter expresses concern about whether the catheter could fall out when the child goes home. What is the nurse's best response?

There is a tiny cuff under the skin that secures the catheter. Explanation: Central venous catheters have a wrinkle-resistant fabric cuff that adheres to the subcutaneous tissue and helps to seal the catheter in place and keep out infection. The cuff does not cushion the tubing, facilitate flushing, or prevent rejection.

A nurse is caring for an infant who just had open-heart surgery and the parents are asking why there are wires coming out of the infant's chest. What is the best response by the nurse?

These wires are connected to the heart and will detect if your infant's heart gets out of rhythm. Following cardiac repair there is always a possibility of arrhythmias. The wires are placed and connected to an external pacemaker. This is done as a preventive measure and can be used if an arrhythmia occurs. Once it is felt the child is in no danger of an arrhythmia, the wires are removed. There is no set time period in which this occurs. The wires do not deliver ongoing electrical shocks to maintain rhythm. This would be done by a permanent pacemaker implanted under the skin. There is no measurement of fluid in the heart by wires. Any measurements would need to be performed by echocardiogram or cardiac catheterization.

A mother is asking for more information about her infant's patent ductus arteriosus (PDA). What would be included in the education?

This is caused by an opening that usually closes by 1 week of age. Explanation: A PDA is caused by an opening called the ductus arteriosus. A PDA usually closes by 1 week of age. If it does not close, the defect is usually fixed by a single surgery or during a cardiac catheterization procedure. IV fluids are not a routine intervention for the PDA. Most of the time a PDA occurs in premature infants.

A child with a congenital heart defect is getting an echocardiogram. How would the nurse describe this test to the parent?

This test will check how blood is flowing through the heart. Explanation: An echocardiogram (echo)= is a noninvasive ultrasound used to assess the heart wall thickness, the size of the heart chambers, the motion of the valves and septa, and the relationship of the great vessels to other cardiac structures. The test evaluates how blood is flowing through the heart and how effective the heart is in pumping. An echo does not check the electrical impulses. This would be done with an electrocardiogram.

Which intervention is the most effective in treating burn wound infections?

Topical antibiotics applied to the wound site

The nursing student identifies which technique as the correct one to use when giving oral medications to an infant?

Use a dropper and slowly inject the liquid into the side of the infant's mouth. Explanation: When giving liquid medication to an infant or child, the nurse should never administer it while the child is flat. Doing so could cause a child to aspirate. The nurse uses the dropper by placing it so the fluid flows slowly into the side of the child's mouth. The nurse should make sure the end of the syringe rests at the side of the infant's mouth to help prevent aspiration as well.

A nurse is preparing to give an intramuscular (IM) injection to an infant. Which site does the nurse identify as mandatory for this administration?

Vastus lateralis muscle For IM injections in infants, the mandatory site for administration is the vastus lateralis muscle of the anterior thigh. This site should be used for all IM injections in infants younger than 7 months of age. After 7 months of age, the ventrogluteal muscle can be used also. The dorsogluteal muscle should not be used for children. The muscles are not fully developed and the sciatic nerve occupies a large portion of this area. The deltoid muscle is used for older children as well as for adults.

You care for a 12-year-old girl with Crohn disease. A primary assessment you would want to make when caring for her would be to note if: a) lung sounds are clear. b) she has a temperature. c) her joints are not swollen. d) she has a headache.

she has a temperature. Because Crohn disease leads to patches of inflammation in the bowel, the temperature increases if more patches become involved.

The nurse is preparing to administer an IV antibiotic to a child. After calculating the recommended dose with the child's weight, the nurse discovers the prescribed dose exceeds the safe dose range in a pediatric drug book. The medication has been given to the child at this dose for 3 days. What action should the nurse take next?

Verify the dose with the prescribing health care provider. Explanation: Medication calculations should always be checked before giving the dose. When a medication dose is found to be outside of the safe dose range, the dose should be verified with the prescribing health care provider. Doses that exceed the recommended range should always be verified, even if they have been given before. The parents did not prescribe this medication. Even if the medication has been given for 3 days, it does not make the dose correct. Calling the pharmacy can only verify if the dose is out of the safe range. The pharmacy did not prescribe the medication nor does it know the child's medical background.

The nurse is caring for a client who has been diagnosed with a tumor in the small intestine that is pressing on the liver. Which type of pain does the nurse anticipate the client will complain of?

Visceral Explanation: Visceral pain is often produced by disease. It usually is diffuse and poorly localized and is described as a deep ache or sharp stabbing sensation that may be referred to other areas. Deep somatic pain typically involves the muscles, tendons, joints, fasciae, and bones and causes dull, aching, or cramping pain. Neuropathic pain usually results in burning, tingling, shooting, squeezing, or spasm-like pain. Chronic pain is defined as pain that continues past the expected point of healing for injured tissue.

A 4-year-old child is being prepared to undergo a bronchoscopy to remove an aspirated pea. The nurse knows that the parents need additional teaching based on which statement?

We will be able to take our child home immediately after the procedure is completed." Explanation: The child will not leave immediately. Procedural complications are not common but may include compromise to the airway such as hemorrhage, pneumothorax, and airway edema. After the procedure, the nurse will need to continue to assess the child's respiratory function and airway patency. Postprocedure complications may include bronchospasm, stridor, desaturation, or respiratory distress. The nurse will observe children carefully the first time they drink after the procedure to assess that the gag reflex is intact and they do not choke. All of the other options are correct.

The nurse is caring for a child who is undergoing peritoneal dialysis. Immediately after draining the dialysate, which action should the nurse should take immediately? -Empty the old dialysate -Weigh the old dialysate -Weigh the new dialysate -Start the process over with a fresh bag

Weigh the old dialysate The nurse should weigh the old dialysate to determine the amount of fluid removed from the child. The fluid must be weighed prior to emptying it. The nurse should weigh the new fluid prior to starting the next fill phase. Typically, the exchanges are 3 to 6 hours apart so the nurse would not immediately start the next fill phase.

In caring for a child with nephrotic syndrome, which intervention will be included in the child's plan of care? -Weighing on the same scale each day -Ambulating 3 to 4 times a day -Increasing fluid intake by 50 ml per hour -Testing the urine for glucose levels regularly

Weighing on the same scale each day The child with nephrotic syndrome is weighed every day using the same scale to accurately monitor the child's fluid gain and loss. The child with nephrotic syndrome is very edematous so increasing fluid intake would be counterproductive to care needed. In nephrotic syndrome the urine is tested for protein, not glucose. Ambulation is important for all but it is not specific to the child with nephrotic syndrome.

The nurse is caring for a 2-year-old girl with suspected vulvovaginitis. The nurse suspects the cause as Candida albicans based on which finding? -White cottage cheese-like discharge -Thin gray vaginal discharge with fishy odor -Foul yellow-gray discharge -Irritation of labia and vaginal opening

White cottage cheese-like discharge White cottage cheese-like discharge indicates C. albicans. Thin gray discharge with a fishy odor points to Bordetella or Gardnerella. Foul yellow-gray discharge indicates Trichomonas vaginalis. Irritation of the labia and vaginal opening is commonly found with poor hygiene.

The caregiver of a child with a history of ear infections calls the nurse and reports that her son has just told her his urine "looks funny." He also has a headache, and his mother reports that his eyes are puffy. Although he had a fever 2 days ago, his temperature is now down to 100℉ (37.8℃). The nurse encourages the mother to have the child seen by the care provider because the nurse suspects the child may have: -a urinary tract infection. -lipoid nephrosis (idiopathic nephrotic syndrome). -acute glomerulonephritis. -rheumatic fever.

acute glomerulonephritis. Acute glomerulonephritis is a condition that appears to be an allergic reaction to specific infections, most often group A beta-hemolytic streptococcal infections such as rheumatic fever. Presenting symptoms appear 1 to 3 weeks after the onset of a streptococcal infection such as strep throat, otitis media, tonsillitis, or impetigo. Usually the presenting symptom is grossly bloody urine. Periorbital edema may accompany or precede hematuria. Fever may be 103℉ to 104℉ (39.4℃ to 40℃) at the onset, but decreases in a few days to about 100℉ (37.8℃). Slight headache and malaise are usual, and vomiting may occur.

The neonatal nurse is assisting the health care provider with a circumcision. Which pain relief method would be most beneficial?

anesthetic cream

A group of students are reviewing information about acute rheumatic fever. The students demonstrate a need for additional review when they identify what as a major Jones criterion?

arthralgia Arthralgia is considered a minor criterion. Carditis is a major criterion. Erythema marginatum is considered a major criterion. Subcutaneous nodules are considered a major criterion.

You care for a 12-year-old girl with Crohn disease. A primary assessment you would want to make when caring for her would be to note if: a) she has a temperature. b) she has a headache. c) lung sounds are clear. d) her joints are not swollen.

she has a temperature. Because Crohn disease leads to patches of inflammation in the bowel, the temperature increases if more patches become involved.

When a child is scheduled for a cardiac catheterization, an important health teaching point for parents is that the:

child will return with a bulky pressure dressing over the catheter insertion area. Explanation: Cardiac catheterization is typically performed with the child awake but using moderate sedation. Only under unusual circumstances will the child need general anesthesia. At the completion of the procedure a pressure dressing will be placed over the catheter insertion site. This is to prevent bleeding. The nurse will monitor this dressing every 15 minutes for the first hour and then every 30 minutes for the second hour. A cardiac catheterization is an invasive procedure and any procedure is frightening to children, especially if their parents are not with them. After the child is fully awake from the procedure the diet can resume.

The nurse is administering an intramuscular injection of an antibiotic to a 3-month-old infant. Which would be the best site for the nurse to give this medication?

vastus lateralis muscle The muscle preferred for intramuscular injections in the infant less than 7 months of age is the vastus lateralis, located on the thigh. The ventrogluteal and deltoid are used in older children to adults. The dorsogluteal is no longer considered suitable for an intramuscular injection due to the risk of injury to the sciatic nerve.

The nurse is providing education to parents of a child with a blood pressure in the 90th percentile. What would be included in the intervention strategies?

he nurse would review the child's 24-hour diet recall. Explanation: With a child in the 90th percentile for blood pressure, diet and physical activity should be the main focus. Blood pressures should be measured but daily measurement is not necessary. Children are not routinely put on beta-blockers, and the child should be allowed to participate in sports if monitored.

A nurse is caring for a child who is grimacing but reports having no pain. What might be the rationale for a child being reluctant to express pain?

fearing getting a "shot" to relieve the pain

After assessing a child, the nurse suspects coarctation of the aorta based on a finding of:

femoral pulse weaker than brachial pulse. Explanation: A femoral pulse that is weak or absent in comparison to the brachial pulse is associated with coarctation of the aorta. Bounding pulse is characteristic of patent ductus arteriosus or aortic regurgitation. A narrow or thread pulse is associated with heart failure or severe aortic stenosis. Hepatomegaly is a sign of right-sided heart failure.

The site most often used when administering a medication using the intradermal route is the:

forearm. Explanation: Intradermal injections deposit medications just under the epidermis. They are most often used for tuberculosis screening and allergy testing. The forearm is the site most often used. The anterior thigh, lateral upper arms, and abdomen are the preferred sites for subcutaneous administration. The deltoid, vastus lateralis and the ventrogluteal are the preferred sited for intramuscular injections.

Noah is an 18-month-old who is brought to the ER with flu-like symptoms. He is diagnosed with pneumonia secondary to aspiration of stomach contents. The nurse explains to the parents that pneumonia is a condition that often occurs secondary to: a) Hirschsprung disease. b) cystic fibrosis. c) gastroesophageal reflux disease. d) inflammatory bowel disease.

gastroesophageal reflux disease. The child with gastroesophageal reflux disease may present with the physical findings of pneumonia secondary to aspiration of refluxed stomach contents.

Noah is an 18-month-old who is brought to the ER with flu-like symptoms. He is diagnosed with pneumonia secondary to aspiration of stomach contents. The nurse explains to the parents that pneumonia is a condition that often occurs secondary to: a) gastroesophageal reflux disease. b) inflammatory bowel disease. c) cystic fibrosis. d) Hirschsprung disease.

gastroesophageal reflux disease. The child with gastroesophageal reflux disease may present with the physical findings of pneumonia secondary to aspiration of refluxed stomach contents.

he new graduate nurse is preparing to administer medication to a 4-year-old client. When would it be appropriate for the supervising nurse to intervene? The new graduate:

had two whole tablets to administer to the child. Explanation: Many children do not have enough coordination to swallow tablets or pills until they are 6 or 7 years of age. Therefore, the supervising nurse would need to intervene. The other actions are correct. The nurse should explain why the medication is being administered. Medications in children are dosed according to body weight (milligrams per kilogram) or body surface area (BSA) (milligrams per square meter). The vastus lateralis is a good location for an IM injection in a 4-year-old child. Reference:

The charge nurse is assisting the new graduate nurse in administering eye drops to a child. The charge nurse would stop the new graduate if which action was observed?

holds the eyelids apart for about 30 seconds Explanation: To prevent the conjunctiva from drying, the nurse should not hold the eyelids apart any longer than necessary. Therefore, the charge nurse would need to stop the new nurse. It is best to use the supine position. Instill the correct number of drops into the conjunctiva of the lower lid. Allow the eyelid to close. Avoid placing the drops directly on the cornea because that can be painful. To prevent the conjunctiva from drying, do not hold the eyelids apart any longer than necessary. After the child has blinked 2 or 3 times, allow the child to sit up.

An adolescent is experiencing severe pain due to a sickle cell crisis. Which medication would be best for the nurse to administer?

hydromorphone For managing severe or acute pain, such as postoperative pain or the pain of a sickle cell crisis, opioids (e.g., morphine, oxycodone, and hydromorphone) are frequently prescribed. NSAIDS and acetylsalicylic acid would not help severe pain.

The nurse is presenting an in-service to a group of nurses who will be working in a dermatology clinic. One participant asks the nurse about a bacterial skin infection that she has seen in children. The nurse most likely is referring to:

impetigo. Impetigo is a superficial bacterial skin infection.

A new nurse is orienting to the newborn nursery and asks the nurse mentor why newborns were not considered to experience pain. How does the mentor explain the rationale used in the past that infants do not experience pain?

incomplete nerve myelination In the past, it was believed that infants do not feel pain because of incomplete myelination of peripheral nerves. Evidence-based practice has shown this not to be true because myelination is not necessary for pain perception. Immature nervous system, age, and assessment tools were not factors in prior beliefs that infants do not perceive pain.

What are possible complications for a child with a vascular access port? Select all that apply.

infection thrombosis hemorrhage air embolism

Most urinary tract infections seen in children are caused by: -hereditary causes. -fungal infections. -intestinal bacteria. -dietary insufficiencies.

intestinal bacteria. Although many different bacteria may infect the urinary tract, intestinal bacteria, particularly Escherichia coli, account for about 80% of acute episodes. Hereditary and dietary concerns are not causes of urinary tract infections.

The nurse is preparing an emergency IV site for a child who has been admitted to the hospital with burns on his arms, legs, and torso. Which IV site would be most appropriate?

intraosseous Explanation: Intraosseous infusion is used in an emergency when it is difficult to establish usual IV access or in a child with such extensive burns that the usual sites for IV infusion are not available. The other sites would not be available in this situation.

The health care provider suspects an infant may have a ventricular septal defect. The parents ask the nurse what diagnostic tests the infant will need to have to determine this diagnosis. For what test(s) should the nurse provide education to the family? Select all that apply.

magnetic resonance imaging (MRI) echocardiogram cardiac catheterization A ventricular septal defect (VSD) is an abnormal opening between the right and left ventricles. MRI or echocardiogram with color-flow Doppler may reveal the opening as well as the extent of left-to-right shunting. These studies also may identify right ventricular hypertrophy and dilation of the pulmonary artery resulting from the increased blood flow. Cardiac catheterization may be used to evaluate the extent of blood flow being pumped to the pulmonary circulation and to evaluate hemodynamic pressures. Neither a CT nor stress test are used in the diagnosis of the VSD. A chest x-ray may also be used to determine if there is enlargement of the heart.

The first method of choice for obtaining a urine specimen from a 3-year-old child with a possible urinary tract infection is: -performing a suprapubic aspiration. -placing a cotton ball in the underwear to catch urine. -placing an indwelling urinary catheter. -obtaining a clean catch voided urine.

obtaining a clean catch voided urine. In the cooperative, toilet-trained child, a clean midstream urine may be used successfully to obtain a "clean catch" voided urine. If a culture is needed, the child may be catheterized, but this is usually avoided if possible. A suprapubic aspiration also may be done to obtain a sterile specimen. In the toilet-trained child, using a cotton ball to collect the urine would not be appropriate.

What would be the most important measure to implement for an infant who develops heart failure?

placing the infant in a semi-Fowler position Semi-Fowlers= Placing an infant with heart failure in a semi-Fowler position reduces the pressure of the abdominal contents against the chest and allows for better lung expansion. Supine= Keeping the infant supine would cause more pressure on the heart and lungs and increase the work of the heart and lungs Calories intake= Infants with heart disease need calories to grow. They are given formula or breast milk which is fortified with extra nutrients. Thus the infant can have an intake of the same amount of fluid but receive extra nutrients.

In caring for an infant diagnosed with pyloric stenosis, the nurse would anticipate that she would: a) prepare the infant for surgery. b) medicate the infant with analgesics. c) change the infant's diet to lactose-free. d) assist in doing a barium enema procedure on the infant.

prepare the infant for surgery. A surgical procedure called a pyloromyotomy (also known as a Fredet-Ramstedt operation) is the treatment of choice for pyloric stenosis.

Which is a priority for the nurse caring for a client with bladder exstrophy? -increasing fluid intake -encouraging voiding -preventing skin breakdown -placing the child in prone position

preventing skin breakdown Prevention of skin breakdown is the priority to prevent infection and the surface from drying out. Encouraging fluids and voiding are not the priority for this client. Prone position is not recommended; the correct position is supine so urine drains freely.

A 4-year-old child is admitted to the hospital for surgery. Before the nurse administers medicine, the best way to identify the child would be to:

read the child's armband. Explanation: A child may answer to the wrong name or deny his or her identity to avoid an unpleasant situation or if scared of the unknown. If the child is avoiding the situation he or she may fail to answer. Using the child's nickname is okay in conversation but it is not a legal identification of the child. To verify the correct identity the nurse should verify the child's armband and the correct name with the child's caregiver. Bar code scanning the child's armband would also be a correct method of identification.

The nurse is caring for a client receiving opioid medication for the treatment of postoperative pain. What are common side effects that the nurse should observe for?

respiratory depression, constipation, and pruritis

A 3-year-old child is scheduled for a surgery to correct undescended testes. For what postoperative consideration would the nurse want to prepare the parents? -the need for complete bed rest for 10 days -some discomfort at the surgery site -a liquid diet for 3 days -the need for maintaining a semi-Fowler position

some discomfort at the surgery site A orchiopexy is the surgical procedure to release the spermatic cord and pull the testes into the scrotum. After the testes are in the scrotum, they are sutured into place to prevent them from returning to the abdominal cavity. This produces a "tugging" or painful sensation. Complete bed rest, a liquid diet, and remaining in a semi-Fowler position are not required as part of the post surgical care.

The caregiver of a child diagnosed with celiac disease tells the nurse that the child has large amounts of bulky stools and what looks like fat in his stools. The clinical manifestation this caregiver is describing is: a) steatorrhea. b) severe diarrhea. c) projectile stools. d)

steatorrhea. Steatorrhea (fatty stools) is a classic symptom of celiac disease. Currant jelly stools are a sign of intussusception. Projectile vomiting is a sign of pyloric stenosis.

What is the only treatment for celiacs disease

strict gluten free diet

An adolescent girl and her caregiver present at the pediatrician's office. The adolescent reports severe abdominal pain. A diagnosis of pelvic inflammatory disease (PID) is made. The nurse notes in the child's chart that this is the third time she has been treated for PID. The most appropriate action by the nurse would be to: -contact the necessary authorities to report a suspected case of sexual abuse. -take the child to a private room and interview her regarding her sexual history and partners. -take the caregiver to a private room and tell her that the child's diagnosis can only come from sexual activity. -talk to the child and caregiver together and explain that the condition is often a result of a sexually transmitted infection and discuss the importance of safe sex practices.

take the child to a private room and interview her regarding her sexual history and partners. Pelvic inflammatory disease can cause sterility in the female primarily by causing scarring in the fallopian tubes that prohibits the passage of the fertilized ovum into the uterus. Adolescents must be made aware of the seriousness of PID, a common result of a chlamydial infection. Be certain to provide the adolescent with a private interview. The adolescent may be extremely reluctant to reveal either social or sexual history especially in the presence of a family member.

A client has been admitted to the emergency department with nausea, vomiting, and severe scrotal pain. These findings indicate what condition? -hydrocele -varicocele -testicular infection -testicular torsion

testicular torsion A hydrocele is a collection of fluid that collects in the fold of the scrotum, requiring no treatment. A variocele is an abnormal dilation of the veins of the spermatic cord. Testicular torsion is evidenced by severe scrotal pain, nausea, and vomiting and is a surgical emergency. Testicular infection is not indicated.

Which statement is the goal of distraction techniques used to control pain?

to divert the child's attention away from the pain through controlled, purposeful behaviors

Which of these laboratory results would be most important for the nurse to assess in a child who has a diagnosis of urinary tract infection? -urinalysis -chemical reagent strip -specific gravity -blood urea nitrogen

urinalysis A urinalysis is one of the simplest tests to reveal kidney function and presence of a urinary tract infection. A chemical reagent strip, specific gravity, and blood urea nitrogen are not the primary tests evaluated for the presence of a urinary tract disease.

The nurse is caring for a 2-year-old postoperative PET client. Which consideration is the most appropriate for this child's developmental stage?

uses words for pain such as owie, boo-boo, or hurt

A 4-year-old child with a urinary tract infection is scheduled to have a voiding cystourethrogram. When preparing the child for this procedure, the nurse would want to prepare the child to: -have a local anesthetic injected prior to the procedure. -drink three glasses of water during the procedure. -void during the procedure. -anticipate a headache afterward.

void during the procedure. At the start of the voiding cystourethrogram, a catheter is inserted into the bladder. The contrast medium is inserted through the catheter into the bladder. Fluroscopy is performed to demonstrate the filling of the bladder and the collapsing of the bladder upon emptying. The assessment of emptying requires the child to void during the procedure so that bladder emptying and urethra flow can be assessed. No anesthetic is required for this procedure. The fluid filling the bladder is inserted via the catheter so no drinking of water is required. A headache following the procedure would not be expected.

A nurse has just admitted a client with symptoms of vulva inflammation, pain, odor, and pruritus. Based on these findings, the nurse could conclude that this client will be diagnosed with which condition? -vulvovaginitis -urinary tract infection -pelvic inflammatory disease -vaginal inflammation

vulvovaginitis Vulvovaginitis is diagnosed with clients experiencing vaginal or vulval inflammation, pain odor, and purititis. Pelvic inflammatory disease and urinary tract infection are not consistent with these symptoms.

The nurse is providing discharge teaching regarding oral fluid rehydration to a mother who brought her child to the clinic because of vomiting over the past 2 days. The child is mildly dehydrated. Which comments by the mother indicated learning occurred? a) "My child can drink milk if they feel like it to help in rehydration." b) "I should be sure my child receives 50 to 100 mL/kg of oral rehydration solution (ORS) over 4 hours." c) "Solutions like Pedialyte are not necessary for mild dehydration." d) "I should not give my child any fluids for 1 to 2 hours after an episode of vomiting." e) "Oral rehydration solutions (ORS) are good sources of fluids for rehydration."

• "I should not give my child any fluids for 1 to 2 hours after an episode of vomiting." • "Oral rehydration solutions (ORS) are good sources of fluids for rehydration." • "I should be sure my child receives 50 to 100 mL/kg of oral rehydration solution (ORS) over 4 hours." In the child with mild to moderate dehydration resulting from vomiting, withhold oral feeding for 1 to 2 hours after emesis, after which time oral rehydration can begin. Tap water, milk, undiluted fruit juice, soup, and broth are NOT appropriate for oral rehydration. Oral rehydration solutions include standard ORS solutions include Pedialyte, Infalyte, and Ricelyte. The recommendation for children with mild to moderate dehydration is 50 to 100 mL/kg of ORS over 4 hours.

The adolescent has been diagnosed with gastroesophageal reflux disease (GERD). Which statements by the teen indicates that adequate learning has occurred? Select all that apply. a) "The omeprazole could give me a headache." b) "I should try to lie down right after I eat." c) "I will probably need a laxative because of the omeprazole." d) "This famotidine may make me tired." e) "It sounds like the physician is reluctant to give me a prokinetic because of the side effects."

• "This famotidine may make me tired." • "The omeprazole could give me a headache." • "It sounds like the physician is reluctant to give me a prokinetic because of the side effects." Famotidine may cause fatigue. Omeprazole can cause headaches. Prokinetics use may result in side effects involving the central nervous system. Omeprazole use more likely will result in diarrhea, not constipation. Children with GERD should not lie down after meals.

Testing is being performed to confirm the presence of Meckel diverticulum. Which findings are consistent with this condition? Select all that apply. a) Platelet count 200,000 b) Hematocrit 37% c) Stool test reveals occult blood. d) White blood cell count 8 g/dL. e) Hemoglobin 9.4 g/dL

• Hemoglobin 9.4 g/dL • Stool test reveals occult blood. Meckel diverticulum is a disorder where there are weaknesses on the intestine resulting in pouchlike areas. Test findings that are consistent with this disorder are anemia and the presence of occult blood. The values listed for white blood cell count, platelet levels, and hematocrit levels are within normal limits.

A child with cirrhosis of the liver is admitted to the acute care facility in preparation for a liver transplant. When completing the physical examination, what would the nurse expect to assess? Select all that apply. a) Jaundice b) Facial erythema c) Fatty stools d) Ascites e) Spider angiomas

• Jaundice • Ascites • Spider angiomas Assessment findings associated with cirrhosis include jaundice, ascites, spider angiomas, and palmar erythema. Fatty stools are associated with celiac disease.


Conjuntos de estudio relacionados

Ch. 28: Pregnancy and Human Development

View Set

Unit 2 Smartbook Assignments and Activities

View Set

Chapter 7: Protective Gear and Sports Equipment

View Set

Completing the Application, Underwriting, and delivering the policy QUIZ

View Set

Last names (Apellidos) customs in the Hispanic world

View Set

Food & Nutrition Unit 6: Water, Vitamins, & Minerals

View Set

AP AMERICAN GOVERNMENT CHAPTER 11

View Set

9.01 Homework Quiz: Moral and Cultural Differences in America

View Set